Você está na página 1de 104

UNIVERSIDADE DE SO PAULO

INSTITUTO DE QUMICA

BIOQUMICA QBQ 0215


FARMCIA NOTURNO

Professores
Nadja C. de S. P. Lardner (nadja@iq.usp.br) coordenadora
Fabio L Forti (flforti@iq.usp.br)
Ricardo J. Giordano (giordano@iq.usp.br)
Mrio Jos Politi (mjpoliti@usp.br)

Monitor

Reginaldo Almeida da Trindade

2009

Apresentao QBQ 0215


Curso de Bioqumica para estudantes do curso noturno da Faculdade de Cincias
Farmacuticas.
Perodo: Segundo semestre de 2009.
Horrio: 19:00 s 23:00 s quartas- e quintas-feiras; 8:00 s 12:00 aos sbados
Local: Salas 9 e 10 do B6 Trreo IQUSP
Neste curso as atividades estaro distribudas em: I-aulas expositivas conforme os
temas constantes no cronograma; II-estudos dirigidos em sala de aula, com superviso dos
professores e monitores (PE); e III- resoluo de exerccios para consolidao dos temas
apresentados, em Grupos de Discusso (GD) que sero realizados aos sbados.
As quartas e quintas-feiras sero apresentadas aulas expositivas, seguidas de
perodos de estudo e exerccios para fixao das matrias. Destes exerccios um ser
selecionado no dia para entrega individual pelos alunos. Este exerccio constar para
avaliao de participao. Desta maneira, o esquema geral para as quartas- e quintasfeiras ser:
19:00 s 20:30 aula expositiva
20:30 s 20:45 intervalo
20:45 s 22:00 perodo de estudos e exerccios
22:00 s 23:00 discusso e Exerccio do Dia
Aos sbados sero apresentadas aulas expositivas de integrao, seguidas de GDs.
Os grupos de discusso consistem de exerccios monitorados para o aprofundamento do
conhecimento pelos estudantes. Os temas da semana sero, ento, avaliados atravs de
Prova tipo GD para ser realizada por grupos de 5-6 alunos; a mdia aritmtica de todas as
Provas GD (GD) compor a nota final, como descrito abaixo. O cronograma para os
sbados ser:
8:00 s 9:45 Aula de Integrao
9:45 s 10:00 intervalo
10:00 s 12:00 Prova GD
Alm das atividades descritas acima, teremos a preparao e apresentao de um
seminrio por grupo, como distribudo no programa. Os temas de seminrio sero
apresentados e distribudos no incio do curso, com um perodo de cerca de dois meses
para a preparao das apresentaes. As apresentaes sero aos sbados, comforme o
cronograma, e devem ser de 25-30 min; sugerimos que os grupos abordem seus temas
2

como propostas de projetos de pesquisa. Os semirios sero avaliados e comporo a nota


final, como descrito abaixo.

Avaliao

O critrio de avaliao ser feito de acordo com a Frmula,


MF = [P1 + P2 + P3 + P4 + (GD + S)/2]/5
Onde:
MF = mdia final
P1-4 = provas 1 a 4
GD = mdia de provas GD
S = nota de seminrio
A mdia mnima para aprovao Cinco.
Para os alunos que perderem alguma prova teremos uma Prova Substitutiva ao final
do curso. Nesta Prova constar toda a matria do semestre.
Bibliografia

Portugus:
Bioqumica Bsica - A. Marzzoco & B.B. Torres - Ed. Guanabara Koogan 2a ed. - 1999.
Princpios de Bioqumica - A.L. Lehninger, D.L. Nelson & M.M. Cox - Ed. Sarvier - 1995.
Bioqumica - J. M. Berg, J.L. Tymoczko & L. Stryer - W.H.Freeman and Company 5th ed. 2002.
Fundamentos de Bioqumica D. Voet, J. G. Voet & C. W. Pratt Artmed Editora- 2000.
Ingls:
Lehninger Principles of Biochemistry - D.L. Nelson & M.M. Cox - Worth Publishers - 3rd
ed. -2000.
Biochemistry - J. M. Berg, J.L. Tymoczko & L. Stryer - W.H.Freeman and Co. - 5th ed. 2002.
Biochemistry - D. Voet & J.G. Voet - John Wiley & Sons - 2004.
Biochemistry - C. K. Mathews & K.E. van Holde - The Benjamin/Cummings Publishing Co.
- 1996.
3

Principles of Biochemistry - H.R. Horton, L.A. Moran, R.S. Ochs, J.D. Rawn & K.G
Scrimgeour Prentice Hall - 1993.
Principles of Biochemistry - G.L. Zubay, W.W. Parson & D.E. Vance - WCB Publishers 1995.

PROFESSORES:
Prof. Dr. Nadja Cristhina de Souza P. Lardner (coordenador)

Bloco 10 Sup., Sala 1065


Fone: 3091-3810 r 242

Prof. Dr. Fabio Luis Forti

Bloco 9 Inf., Sala 908


Fone: 3091-3810 r 216

Prof. Dr. Ricardo Giordano

Bloco 10 Inf., Sala 1011


Fone: 3091-3810 r 233

Prof. Dr. Mrio Jos Politi

Bloco 12 Sup., Sala 1258


Fone: 3091-3877

MONITORES:
Reginaldo Almeida da Trindade

e-mail:registrindade@gmail.com

Cronograma

2O SEMESTRE FARMCIA 2008


QBQ 215

Agosto
19/08 P
20/08 P
22/08 P
26/08 P
27/08 P
29/08 P
Setembro
02/09 P
03/09 P
05/09 R
09/09 P
10/09 P
12/09 P
16/09 P
17/09 P
19/09 P
23/09 P
24/09 P
26/09 P
30/09 P
Outubro
01/10 P
03/10 P
07/10 P
08/10 P
10/10 P
14/10 15/10 17/10 21/10 F
22/10 F
24/10 F
28/10 29/10 F
31/10 F
Novembro
04/11 F
05/11 F
07/11 F
11/11 R
12/11 R
14/11 P

dias
quarta
quinta
sbado
quarta
quinta
sbado
quarta
quinta
sbado
quarta
quinta
sbado
quarta
quinta
sbado
quarta
quinta
sbado
quarta
quinta
sbado
quarta
quinta
sbado
quarta
quinta
sbado
quarta
quinta
sabado
quarta
quinta
sabado
quarta
quinta
sbado
quarta
quinta
sabado

Mdulos
Apresentao / gua, tampes, pH do sangue
Aminocidos: Funes e Propriedades
Aula integrao & GD 1
Estrutura e funo de Protenas I
Estrutura e funo de Protenas II
Aula integrao & GD 2
Enzimas - conceitos gerais
Cintica enzimtica
Aula integrao & GD 3
cidos Nuclicos e Estruturas
Acares e Polissacardeos
Aula integrao & GD 4
Prova 1
Lipdeos e membranas
Aula integrao & GD 5
Colesterol, partculas carreadoras e sais biliares
Bioenergtica e ATP
Aula integrao & GD 6
Gliclise
Ciclo de Krebs
Aula integrao & GD 7
Fosforilao Oxidativa
Prova 2
GD 8 e Seminrios (2)
Semana da Farmcia (no haver aula)
Semana da Farmcia (no haver aula)
Semana da Farmcia (no haver aula)
Ciclo das Pentoses
Neoglicognese
GD 9 e Seminrios (2)
Dia do funcionrio pblico (no haver aula)
Metabolismo de Glicognio
GD 10 e Seminrios (2)
cidos Graxos - Degradao
cidos Graxos - Sntese
GD 11 e Seminrios (2)
Metabolismo de Amino cidos
Prova 3
GD 12 e Seminrios (2)
6

18/11 R
19/11 R
21/11 R
25/11 R
26/11 N
28/11 R
Dezembro
02/12 N
03/12 N
05/12 N
09/12 N
10/12 N
12/12 N
16/12 N
17/12 N

quarta
quinta
sbado
quarta
quinta
sbado
quarta
quinta
sbado
quarta
quinta
sbado
quarta
quinta

Ciclo da Uria
Metabolismo de Purinas e Pirimidinas
GD 13 e Seminrios (2)
Fotossntese
Integrao do Metabolismo
GD 14 e Seminrios (2)
Fluxos de materiais em Humanos,Transporte em Membranas
Diabetes e Hormnios
GD 15 e Seminrios (2)
Metabolismo Muscular
Transduo de Sinais
Prova 4
Aula de integrao e GD 16
Prova substitutiva

P = Politi
F = Fabio
R= Ricardo
N = Nadja

Temas para os Seminrios:


1. As reaes qumicas e a origem da vida
2. Importncia da estrutura das protenas para as propriedades fsicas de materiais
biolgicos
3. Enzimas como alvos farmacuticos
4. Alcoolismo
5. Obsidade e desnutrio
6. Sndrome Metablica e diabetes
7. Osteoporose e Hipercalcemia
8. O metabolismo e a pirmide alimentar
9. Doenas do metabolismo de purina/pirimidinas
10. Ictercias
11. Hiperlipidemia e Doenas Cardiovasculares
12. O nitrognio e a vida
13. Fotossntese e o impacto biolgico do ciclo de carbono
14. Cancer e estratgias anti-neoplasicas
15. Membranas como alvos farmacuticos
16. Doenas mitocondriais

H2O e Sistemas TAMPO

1. A molcula de gua, H2O, apresenta um ngulo de 104,5 graus entre as duas ligaes O-H,
dando-lhe um carter altamente polar. Alm disso, o tomo de O possui 2 pares de eltrons
livres, permitindo a formao de ligaes (ou pontes) de H entre molculas vizinhas. Esta
estrutura d gua propriedades fsicas e qumicas de enorme importncia biolgica.
2. A gua se ioniza atravs de uma reao cido-base:
H2O + H2O

H3O++ OH-

A reao cido-base se caracteriza pela troca de prtons entre pares conjugados de cidos e
bases. A gua pode se comportar como cido e como base:
H3O+ + A-

AH + H2O

BH + OH-

B + H2O

Estas so reaes de equilbrio, s quais correspondem constantes de equilbrio definidas.


Por exemplo: K = [H3O+] [A-]
[AH] [H2O]
K mede a afinidade relativa das bases, de cada par cido-base conjugados (AH/ A- e H3O+/
H2O), por prtons. Fala-se comumente em constante de dissociao de um cido (Ka),
significando: Ka = K [H2O] = [H+] [A-], onde [H2O] essencialmente constante (55 M).
[AH]
3.

[H ] a concentrao hidrogeninica e os valores de [H+] para a maioria das solues so


muito baixos e difceis de serem comparados. Um valor mais prtico conhecido como pH:
pH = - log [H+].
como

1/[H+] = 1/K x [A-]/[AH]

pode-se obter

pH = - logK + log [A-]/[AH]

por analogia

- log K = pK

pH = pK + log [A-]/[AH]

Conclui-se que pK numericamente igual a pH da soluo na qual as concentraes


molares do cido e sua base conjugada so iguais (ie log [A-]/[AH] = 0).
A igualdade pH = pK + log [A-]/[AH] conhecida como Equao de HendersonHasselbach.
4. cidos so classificados de acordo com sua fora relativa, ou seja, de acordo com sua
capacidade de transferir um prton para a gua. cidos com constantes de dissociao
menores do que aquela de H3O+ (que, por definio, igual a 1 em solues aquosas (v se
consegue confirmar porqu!)) so s parcialmente ionizados em solues aquosas e so
conhecidos como cidos fracos (K < 1). J os cidos fortes tm constantes de dissociao
maiores que a de H3O+, sendo quase completamente ionizados em solues aquosas (K>1).
5. Tampes so sistemas aquosos que tendem a resistir a variaes no seu pH quando
pequenas quantidades de cido (H+) ou base (OH-) so adicionadas. Um sistema tampo

consiste de um cido fraco (o doador de prtons) e sua base conjugada (o aceptor de


prtons). comum encontarr os seguintes smbolos para representar um cido (AH ou BH+)
-

e sua base conjugada (A ou B:)


6. A adio de cido forte (H+) ou base forte (OH-) a uma soluo aquosa de um cido fraco,
por exemplo, cido actico (pKa = 4,76), causa pequenas variaes de pH, se a soluo
estiver a um pH prximo do pK do cido. Este comportamento define um tampo cido-base.
7. O pH do sangue mantido atravs de um sistema de reaes envolvendo a Hemoglobina e
o CO2, ie o on HCO3-.
Exerccios 1
1) Defina cidos e bases no conceito de Brnsted, mostrando exemplos.
2) a) Qual o pH das solues 0,1 M dos cidos fortes HCl e HNO3? b) Usar a equao HendersonHasselbach para calcular o grau de dissociao dos cidos fracos i) H2S (Ka=1x10-7) e ii) cido
actico (Ka=2x10-5) em solues 0,1 M. Qual o respectivo pH dessas solues?
3) Esquematize a curva de titulao de 1 L de uma soluo de 0,1 M H3PO4 com uma soluo de 10
M NaOH, colocando pH (eixo y) em funo de volume de base adicional (eixo x). Indicar os
pontos na titulao (volumes de NaOH) em que o pH equivale cada um dos pKas do cido.
4) Indique como se pode preparar 1 L de um tampo a pH=7,0, capaz de manter o pH estvel com
adio de 10 mL de HCl 0,1M, dispondo-se das solues:
a) 1M H3PO4
b) 1M cido actico
c) 1M NaOH
5) Desenhe a estrutura do gelo, mostrando pontes de hidrognio entre molculas de gua. O que
acontece quando o gelo derrete? Porque a gua lquida 4oC mais densa do que o gelo 0oC?
6)Discuta as propriedades de gua em comparao a outros solventes e a molculas isoeletrnicas.
7) Desenhe a estrutura do NaCl no estado slido e tambm no estado aquoso, neste ltimo,
destaque suas interaes com gua.

8) Discuta como o sistema tampo do sangue mantm o pH estvel em condies de


acidose e de alcalose.

10

AMINOCIDOS
1. Aminocidos, bases purnicas e pirimidnicas, nucleosdeos e nucleotdeos, hexoses (como
glicose), so componentes monomricos dos principais polmeros biolgicos, ou seja,
protenas, cidos nuclicos (DNA e RNA) e polissacardeos (glicognio, amido e celulose).
Aminocidos, bases, nucleosdeos e nucleotdeos so em geral muito solveis em gua e
possuem grupos funcionais que participam em reaes cido-base. Glicose tambm
altamente solvel em gua, mas no participa em reaes cido-base.
i. H 20 aminocidos que compem protenas (Tabela 1), todos mostrando a frmula geral:
R
+

H3N

COO-

on dipolar ou zwitterion encontrado em gua pH 7

H
2. Aminocidos podem ser agrupados em classes com base nas propriedades dos seus grupos
radicais (R), em particular sua polaridade ou tendncia de interagir com gua em pH
biolgico ( 7,0).
3. Todos os aminocidos livres comportam como cidos poliprticos. Quando um aminocido
cristalino dissolvido em gua, ele pode agir como um cido ou como uma base. O grupo
carboxlico mostra um pK em torno de 2,0, enquanto o grupo amino tem um pK entre 9,0 e
10,0. Portanto, no pH fisiolgico (pH 7,0), a maioria das molculas de todos os aminocidos
est na forma de ons dipolares (zwitterions). Chama-se pI de um aminocido o pH da
soluo na qual suas molculas possuem carga lquida nula. Na cadeia lateral (-R) os
aminocidos apresentam grupos funcionais, entre os quais existem grupos cido-base.
4. O carbono dos aminocidos, excetuando-se a glicina, assimtrico, fazendo com que
estas substncias tenham atividade ptica e, portanto, apresentem pares de ismeros
pticos.

Exerccios 2
1) Quais dos aminocidos tm dois carbonos quirais e qual deles no possui isomeria ptica?
2) Mostre porque a seguinte forma no-inica de um aminocido no pode ser encontrada em
soluo aquosa.
R
H2N

COOH

11

3) O etanol no tem carter cido em gua, enquanto fenol e cido actico se dissociam em soluo
aquosa, sendo o cido actico (pK=4,8) mais forte que o fenol (pK=10). Como se pode explicar o
comportamento destes trs compostos em gua a partir de suas estruturas moleculares?
4) Esquematize a curva de titulao da glicina com NaOH a partir de pH=1 e do cido asprtico com
HCl a partir de pH=11. Coloque o pH na ordenada e, na abscissa, a quantidade de equivalentes
de cido ou base forte.
5) a) Quais os pontos isoeltricos de: glicina (pKs=2,5 e 9,5), cido asprtico (pKs=2,5; 4,0 e 9,5),
lisina (pKs=2,5; 9,5 e 10) e histidina (pKs=2,5; 6,0 e 9,5)? b) Calcular as cargas lquidas
(aproximadas) do cido asprtico, lisina ou histidina nos seguintes pHs: pH 1, pH 8, pH 11.
6) Tentar classificar os aminocidos em termos da natureza qumica dos seus grupos radicais: a)
ionizveis ou no ionizveis, b) cidos ou bsicos, c) polares ou no polares, d) hidroflicos ou
hidrofbicos, e) alifticos ou aromticos, f) lineares ou ramificados e g) pequenos e grandes.
7) Na Tabela 1 indicar: a) O cdigo de letra nica para cada aminocido e b) os pKR dos
aminocidos com grupos radicais ionizveis.

12

Tabela 1: Estrutura dos 20 aminocidos encontrados em protenas

13

ESTRUTURA PRIMRIA DE PROTENAS


1. A descrio da estrutura das protenas dividida em quatro nveis de organizao: estrutura
primria, secundria, terciria e quartenria.
2. A estrutura primria se refere seqncia de aminocidos que compem a protena. Tratase, portanto, da estrutura de ligaes covalentes. A principal ligao covalente entre
aminocios a ligao peptdica. Os aminocidos podem formar polmeros atravs da
ligao do grupo carboxila de um aminocido com o grupo amino de outro. Esta ligao
carbono-nitrognio chamada ligao peptdica, obtida por excluso de uma molcula de
gua. Quimicamente, a formao da ligao peptdica pode ser representada pela seguinte
equao:

Esta reao, como esta escrita, jamais ocorre nos seres vivos. A unio dos aminocidos por
ligao peptdica no feita por reao direta entre eles, mas atravs de um complexo
aparato de sntese protica, que inclui ribossomos, cidos ribonuclicos, vrias protenas e
enzimas num processo chamado traduo. A equao mostra apenas o resultado liquido do
processo.
3.

As propriedades da ligao peptdica impem restries ao dobramento do polmero formado. A


ligao peptdica, apesar de ser representada por um nico trao de ligao, tem caractersticas
intermediarias entre uma ligao simples e uma dupla ligao, devido s interaes entre duas
formas de ressonncia.

4.
A conseqncia desse carter parcial de dupla ligao que no h possibilidade de rotao
em torno da ligao peptdica. Assim sendo, os quatro tomos dos grupamentos que
participam da ligao peptdica ficam dispostos em um plano rgido, constituindo o que se
costuma chamar de grupo peptdico ou unidade peptdica (vide retngulos) Notar tambm
que os dois carbonos alpha (C) vizinhos de cada ligao peptdica tambm se encontram o
plano.

14

Marzzocco & Torres, Bioqumica Bsica.

O polmero formado pode, portanto, ser visualizado como uma cadeia constituda por
unidades planares (unidades peptdicas), unidas entre si com uma articulao flexvel: o
carbono . Esta cadeia chama-se cadeia polipeptdica. As protenas podem ser formadas por
uma ou mais cadeias polipeptdicas.
4.

Todavia, existem pontos de dobramento entre as unidades peptdicas rgidas, graas a


possibilidade de rotao em torno das ligaes com o carbono alfa (N-C e C-C), que so
ligaes efetivamente simples (vide figura acima). Estas ligaes so chamadas phi () e psi
() respectivamente.

5.

A cadeia polipeptdica pode ser dividida entre a cadeia principal e as cadeias laterais
(grupos R) ligados aos carbonos alfa.

Exerccios 3
1) Defina estrutura primria, secundria, terciria e quaternria de uma protena, dando exemplos.
2) Esquematize a estrutura de uma ligao peptdica.
3) a) Desenhar o tripeptdeo Ala-Asp-His. b) Calcular o seu pI. c) Calcular sua carga lquida em pH
1, pH 6 e pH 12.

15

4) Com os dados abaixo, defina a seqncia do peptdeo analisado: a) hidrlise cida total resultou
em: Arg, Tyr, Leu, Ala, Glu Lys, Ser e Pro; b) dansilao e hidrlise produziram: dansil-Leu; c) dois
ciclos consecutivos de degradao de Edman liberaram, respectivamente Leu e Tyr; d) tripsina
liberou 2 peptdeos cujas composies, aps hidrlise cida total, foram, respectivamente (Tyr, Leu,
Arg) e (Ser, Glu, Pro, Ala Lys); e) carboxipeptidase A no liberou nada, mas carboxipeptidase C
liberou Ser; f) endopeptidase V8 liberou o tripeptdeo Lys-Pro-Ser e um pentapeptdeo que, tratado
com carboxipeptidase C, liberou Glu.

5) Mostre a reao de xido-reduo da cistena que importante na estrutura de


peptdeos.

16

ESTRUTURA SECUNDRIA E TERCIRIA DE PROTENAS

1. A estrutura secundria definida pela conformao local do esqueleto de ligaes peptdicas


que compe o eixo da protena. Esta conformao local pode ser explicitamente expressa
atravs dos ngulos phi () e psi (). Em geral, certas combinaes de ngulos phi () e psi
() so permitidas enquanto outras no so permitidas devido a impedimentos estricos
entre tomos de grupos vizinhos. Este princpio pode ser resumido num diagrama de
Ramachandran (Figura 1).

Figura 1: Diagramas de Ramachandran. Esquerda: Estruturas secundrias correpondentes s


combinaes estericamente permitidas para angulos phi e psi. Direta: ngulos observados para
todas as ligaes em 12 protenas com estruturas de alta resoluo determinadas por
cristalografia.

17

Figura 2: -hlice.

Figura 3: Folha pregueada.

2. H duas estruturas secundrias principais: -hlice (Figura 2) e folha pregueada (Figura 3),
que so estruturas organizacionais regulares e repetitivas. Estas duas estruturas podem ser
caracterizadas por combinaes de angulos phi e psi (Figura 1) adotadas pela cadeia principal.
Alm de -hlice e folha , as protenas globulares mostram tambm alas de formas definidas,
mas irregulares e no repetitivas.
5. A estrutura terciria descreve o arranjo tridimensional da cadeia principal da protena, incluindo a
disposio espacial das cadeias laterais dos aminocidos. H muitas possibilidades de arranjos
tridimensionais para a estrutura terciria das protenas.
a. As propriedades bioqumicas e biolgicas de uma protena so determinadas pelo arranjo
tridimensional de sua cadeia, isto , pela sua estrutura terciria. Logo, nas condies
fisiolgicas a protena adquire uma estrutura terciria bem definida e necessria sua
funo, que conhecida como estrutura nativa. O desarranjo da estrutura terciria leva
perda de funo da protena, processo que genericamente chamado de desnaturao.
b. Em protenas pequenas da estrutura primria define a estrutura terciria nativa da protena.
Nestes casos os processos de desnaturao e renaturao da estrutura da protena so
reversveis. A estrutura nativa a conformao da protena de menor nvel de energia livre
(G) e alcanada espontaneamente (processo exergnico). O exemplo clssico desse
comportamento dado pela protena Rnase A, uma enzima que no seu estado nativo
catalisa a hidrlise de RNA. Para protenas grandes o processo de desnaturao
irreversvel e o fenmeno de alcance da conformao nativa complexo e ainda mal
entendido.

18

c. A estrutura tridimensional das protenas mantida por ligaes fracas como pontes de H,
ligaes inicas e interaes hidrofbicas. A exceo a ponte de dissulfeto (-S-S-) que,
apesar de covalente, importante na manuteno da conformao nativa de protenas.
d. Protenas possuem muitos grupos ionizveis atravs de reao cido-base, cujos pKs variam
enormemente. O pI de uma protena definido como pH da soluo na qual a carga lquida
da molcula de protena nula.
4. Existem muitas maneiras diferentes para apresentar estruturas tridimensionais de protenas.

Estrutura de mioglobina de baleia, uma protena globular tpica

Topografia
de superfcie

Fita (azul = H)

modelo space-filling
19

6. A Hb e a Mb so protenas com funes parecidas. Ambas apresentam afinidade pelo O2


molecular. A HB o carreador do O2 pelo sistema arterial e capilares enquanto a Mb
seqestra o O2 em nvel do tecido muscular. Servindo como depsito para a contrao
do msculo em aerobiose. A curva de interao Hb/O2 tem carter sigmoidal (alostrica)
emquanto a da Mb hiperblica. Esta diferena permite um controle fino da troca de O2
entre artrias e msculo. Em paralelo a curva de afinidade O2/Hb entre a me e o feto
apresenta maior afinidade pela Hb fetal permitindo a troca eficiente de O2 para o feto.
7. Efeito Bohr.
Exerccios 4
1) Distinga estrutura secundria e terciria de uma protena. D exemplos.
2) Descreva -hlice e folha pregueada. Aponte as diferenas essenciais entre estas formas de
estrutura secundria encontradas em peptdeos.
3) Discuta os dois diagramas de Ramachandran apresentados na Figura 1 e relacione-os com as
estruturas apresentadas nas Figuras 2 e 3.
4) Descreva a experincia clssica de Anfinsen com a enzima ribonuclease A, indicando sua
concluso principal. Qual o papel das pontes de dissulfeto na manuteno da estrutura nativa
(terciria) da ribonuclease? Conceitue estrutura nativa e desnaturao de protenas, mostrando
o que isso tem a ver com a atividade enzimtica da ribonuclase A. Que funo termodinmica
promove espontaneamente a transio da ribonuclease de desnaturada para nativa?
5) Duas protenas, apesar de terem diferenas quanto a alguns de seus aminocidos, so capazes
de desempenhar a mesma funo. Explique como isto possvel.
6) Pesquisar informaes sobre a estrutura de hemoglobina. Descrever a sua estrutura terciria e
quartenria. Descrever as mudanas na estrutura quartenria que acontecem devido ligao de
oxignio.
7) O que efeito hidrofbico e qual o seu papel na manuteno da estrutura terciria das protenas?
Qual o fator preponderante no efeito hidrofbico: o entlpico ou o entrpico? Explique
qualitativamente sua resposta.
8) Discuta porque uria e cloreto de guanidina desorganizam a -hlice.

20

CINTICA ENZIMTICA

1.

Enzimas so catalisadores biolgicos cuja natureza qumica proteica. A natureza proteica das
enzimas lhes proporciona alto grau de especificidade.

2.

A grande maioria das reaes biolgicas no ocorre, ou ocorrem a velocidades baixssimas nas
condies fisiolgicas de pH e temperatura. Logo, as reaes biolgicas, em geral, necessitam
de catlise para ocorrer, isto , necessitam de enzimas. Para cada reao h uma enzima
especfica.

3.

Na reao genrica A B a direo espontnea da reao dada pela variao de energia


livre, .G0, conforme esquematizado no grfico da Figura 5.

Energia
Livre (G)

*
G10#

Estado de transio da
reao no catalisada

G0#-1
*

Estado Inicial
(S)
(Reagentes)

Estado de transio da
reao catalisada

G0#1cat
0

G0#-1cat

Estado Final
(P)
(Produtos)

Coordenada de Reao
Figura 6. Variao de energia livre (G) na reao genrica A B.
G0 uma constante que se relaciona com a constante de equilbrio da reao pela expresso G0=2.3 RTlogK. Por outro lado, as velocidades das reaes AB e BA ou, respectivamente,
as constantes de velocidade k1 e k-1 no dependem do G0 da reao, mas dos, respectivos,
G10 e G-10, que por sua vez s dependem da energia livre (G) do estado de transio
(energias de ativao). A enzima (catalisador) no muda o G0 da reao, pois
catalisadores no interferem com os estados inicial e final das reaes, mas mudam o
caminho da reao e, por conseqncia diminuem a energia do Estado de Transio.

4.

Uria uma substncia muito estvel em gua, mas que pode ser rapidamente decompostas
por hidrlise se a reao for catalisada pela enzima urease:

21

H2N

UREASE

C=O + H2O

CO2 + 2 NH3

H2N
Trata-se de reao de primeira ordem, onde v=k1[uria], apesar da equao estequiomtrica
indicar a existncia de 2 reagentes. Esta reao pode ser acompanhada em tubo de ensaio no
laboratrio. As Tabelas 3 e 4 mostram resultados obtidos na prtica.

Tabela 3. Cintica da enzima urease.


Tubo no

Tempo (minuto)

NH3(
moles)

0.084

0.168

0.252

0.336

10

0.420

Concentrao da uria: 5 mM; Concentrao da urease: 0,1 g/mL;


Volume de reao: 1 mL; Temperatura: 30oC.

Os dados da Tabela 3 mostram que a velocidade da reao constante ao longo do tempo


estudado. J os dados da Tabela 4 mostram variaes relativamente complexas da velocidade
de reao em funo da concentrao da uria para um perodo de 10 minutos de reao.
Os dados da Tabela 4 permitem medir experimentalmente duas constantes importantes das
reaes enzimticas Vmax (velocidade mxima) e Km (constante de Michaelis) atravs da
equao v = Vmax[S] / (Km + [S]).

Tabela 4. Cintica da enzima urease.


Tubo n

Uria (mM)

Urease (
g)

NH3 (
moles)

2,5

0,1

0,21

5,0

0,1

0,42

10

0,1

0,59

22

15

0,1

0,67

25

0,1

0,73

50

0,1

0,78

100

0,1

0,79

200

0,1

0,78

200

0,00

Os significados de Vmax e Km so definidos no modelo de cintica enzimtica proposto por


Michaelis e Menten no incio do sculo passado onde ES um complexo enzima substrato
formado antes de converso do substrato em produtos.

E + S

k1
k-1

ES

kcat

E + P

A derivao da equao Michaelis Menten:


v = Vmax[S] / (Km + [S]) = kcat[Et][S] / (Km + [S])
apresentada na prxima pgina.

23

Velocidade
naquela [S]

Velocidade mxima

Kdiss aparente do
Complexo enzima-substrato

Concentrao
do substrato

Frao de Etot na forma de ES =


[S]/(Kdiss + [S])

24

5.

Substncias que reduzem a atividade de uma enzima so chamadas inibidores. Em termos


gerais, inibidores podem atuar em vrias maneiras. Aqui vamos focalizar em inibidores que
ligam reversivelmente com a enzima com constantes de dissociao KI. Estes tipos de
inibidores podem atuar em duas maneiras diferentes: a) Eles podem competir com o substrato
para o mesmo stio de ligao na superfcie da enzima livre. Neste caso so chamados
inibidores competitivos ou b) Eles podem ligar em outro stio na enzima livre (E) e/ou no
complexo enzima-substrato (ES). Estes inibidores so chamados inibidores mistos/nocompetitivos se podem ligar a E e ES e so chamados acompetitivos se ligam somente ao
complexo ES.

6.

A presena de um inibidor competitivo se manifesta em uma mudana no valor do Km:


Km obs = Km(1+[I]/KI) = Km

7.

onde

(1+[I]/KI)

A presena de um inibidor misto/no-competitivo se manifesta em uma mudana nos valores do


Km e no valor do Vmax:
Km obs = Km(1+[I]/KI)/(1+[I]/KI) = Km
Vmax obs = Vmax /

8.

A presena de um inibidor acompetitivo se manifesta em uma mudana nos valores do Km e no


valor do Vmax:
Km obs = Km / (1+[I]/KI) = Km
Vmax obs = Vmax /

Exerccios 5
1) As velocidades de uma reao enzimtica foram determinadas para diversas concentraes de
substrato, conforme a tabela abaixo:
[S] (M)

V (mol/L.min)

22

10

39

20

65

50

102

100

120

200

135

Os grficos de, respectivamente, V em funo de [S] e 1/V em funo de 1/[S] podem servir para
determinar Km e Vmax? Como?

25

2) Numa reao enzimtica, o valor de Vmax, mas no o de Km diretamente proporcional


concentrao da enzima? Justifique.
3) A velocidade inicial de uma reao enzimtica em funo da concentrao do substrato S, na
ausncia e na presena dos inibidores A e B segue os dados da tabela abaixo:

[S] (M)

VELOCIDADE (
MOL/L X MIN)
SEM I

Com Inibidor A

Com Inibidor B

1,25

1,72

0,98

1,01

1,67

2,04

1,17

1,26

2,5

2,63

1,47

1,72

5,0

3,33

1,96

2,56

10,0

4,17

2,38

3,49

a) Qual a classe dos inibidores A e B?


b) Determine Vmax e Km na ausncia e presena dos inibidores.
4) Utilizando-se dos valores de Km e Vmax determinados nas questes 1 e 3, esquematize num
mesmo grfico, para as duas reaes, V em funo da concentrao de substrato, expressa em
mltiplos de Km. No eixo dos Y ajuste arbitrariamente as escalas para cada reao fazendo
coincidir os pontos de V = Vmax. Como so as curvas para duas reaes? Justifique o resultado.

5) O que so enzimas alostricas? Defina utilizando-se de grficos esquemticos de V em


funo de [S], compare uma enzima michaeliana (da questo 4) com uma enzima
alostrica positiva e com uma enzima alostrica negativa.

26

MECANISMOS DE CATLISE ENZIMTICA

1. Catlise cido/base catlise por transferncia de protons. A catlise cida um


processo no qual a transferncia parcial de prtons de um cido para o estado de transio
diminui a energia livre do estado de transio de uma reao. A reao pode ser tambm
estimulada por uma catlise bsica se a taxa de reao aumentar com a abstrao de um
prton por uma base. Algumas reaes podem ser sujeitas simultaneamente a ambos os
processos, caracterizando uma catlise cido-base. Em reaes catalisadas por enzimas os
cidos e bases catalisadores so grupos especficos ionizveis da enzima localizados no seu
stio ativo/stio cataltico. A mutarrotao da glicose (Figura 6) e a catlise da ribonuclease
pancretica bovina A (RNase A) (Figura 7) so exemplos de catlise cido-base.

Figura 6. Mutarrotao da glicose.

27

Figura 7. Catlise da ribonuclease pancretica bovina A (RNase A).

28

2. Catlise covalente envolve a acelerao da taxa de reao atravs da formao transiente


de uma ligao covalente substrato-catalisador. A descarboxilao do acetoacetato um
exemplo deste processo:

No primeiro estgio desta reao, a amina faz um ataque nucleoflico ao grupo carbonila do
acetoacetato formando uma base de Schiff (ligao imina).

OH

O tomo de nitrognio protonado do intermedirio covalente atua como um receptor de eltrons


reduzindo assim o carter de alta energia do enolato. A catalise covalente possui estgios
nucleoflicos e eletroflicos. A catlise covalente pode ser dividida conceitualmente em trs
estgios:
1) A reao nucleoflica entre o catalisador e o substrato formando uma ligao
covalente.
2) A perda de eltrons do centro da reao pelo catalisador agora eletroflico.
3) A eliminao do catalisador que uma reao essencial para retornar ao estgio 1.

Exerccios 6
1) Examine a reao de hidrlise de RNA catalisada pela RNase A para verificar que se trata de um
mecanismo de catlise cido-base.
2) Faa o grfico da velocidade de uma reao enzimtica em funo do pH para uma enzima
estvel entre pHs 3 e 12, considerando que o substrato no possui grupos ionizveis e a

29

atividade enzimtica exige no centro ativo uma carboxila (pKa = 5) desprotonada e um grupo
amino (pKa = 9) protonado.
3) Definir catlise eletrosttica. Procure um exemplo de uma enzima que utiliza esta estratgia.
4) Descrever o mecanismo empregado pelas serina proteases (tripsina, quimiotripsina, elastase, etc)
para hidrolisar ligaes peptdicas. Descrever todas as etapas da reao. Quais tipos de catlise
so empregados em cada uma das etapas?

30

CIDOS NUCLEICOS
Tendo em vista que no curso do prximo semestre, dedicado Biologia Molecular, neste modulo
daremos nfase nicamente aos aspectos estruturais de RNA e DNAs. Conforme j bem
estabelecido estes polmeros (polieletrlitos) so constitudos por um corrimo que consiste de
riboses e deoxiriboses grupo fosfato, e pendente `a estes encontram-se as bases pricas e
pirimidnicas. Nos RNAs estas bases so C, T, A e U j nos DNAs as bases so C, T, A e G.
1) A estrutura do DNA de dupla fita e do RNA fita simples.
2) Nucleosdeos so denominados os monmeros compostos de purinas ou pirimidinas aos
acares ribose e deoxiribose (deoxinucleosdeo).
3) Nucleotdeos (deoxinucleotdeos) so ster de fosfato de nucleosdeos (deoxinucleosdeos).
4) A fita de DNA em -hlice apresenta as denominadas cavernas (grooves) maior e menor. Stios
estes de ligao de molculas.
5) as bases constituem espcies de degraus, sendo as interaes entre T-A estabilizada por 2
pontes de H e C-G por trs pontes.
6) O teor de CG e AT determina a temperatura de fuso das cadeias ie a temperatura de separao
das cadeias. Esta temperatura acorre abruptamente.
7) O DNA encontra-se altamente enovelado no ncleo da clula.
8) O fenmeno de hipercromismo ajuda detectar a temperatura de melting das cadeias e permite
fazer a hibridizao de cadeias.
9) Molculas de DNA so extremamente longas e frgeis. A simples sonicao leva fragmentao
em vrias sub-partculas.

Exerccios 7
1) Esboce as estruturas do DNA e do RNA.
2) Por que o DNA chamado cido DESOXIribonucleico e o RNA chamado somente ribonucleico?
3) Como deve ser a viscosidade de solues de DNA enovelado e aps a fuso?

31

4) Qual deve ser o efeito de sais na questo 3?


5) Considerando seja um DNA linear ou circular como possvel enovelar o DNA em tal
dimenses?

32

ACARES E POLISSACARDEOS: ESTRUTURA E FUNO

1. Os carboidratos so compostos que apresentam a frmula emprica (CH2O)n (n> ou = 3),


sendo funcionalmente poliidroxialdedos ou poliidroxicetonas. Os carboidratos mais simples
so os monossacardeos, que se apresentam na formas de aldoses ou cetoses, conforme o
grupo funcional carbonlico que possuem, isto , respectivamente, aldedo ou cetona. H
duas trioses: o gliceraldedo, uma aldotriose, e a diidroxiacetona, uma cetotriose (Figura 8).
O gliceraldedo apresenta um carbono (C2) assimtrico, dando origem a dois ismeros
opticos, as formas D e L (Figura 9). J a diidroxiacetona no possui C assimtrico e, por isso,
no mostra esse tipo de isomeria. Os outros monossacardeos podem ser derivados pelo
crescimento da cadeia destas duas trioses. A Figura 10 mostra a famlia D derivada do Dgliceraldeido, cujas frmulas estruturais planares obedecem as regras de Fisher.

Figura 8. Gliceraldedo e diidroxiacetona.

Figura 9: Carbono quiral ou


carbono assimtrico.

33

Figura 10. Famlia D derivada do D-gliceraldedo.

Figura 11. Ciclizao da D-glicose.

34

O aumento da cadeia do monossacardeo leva ao aparecimento de novos Cs assimtricos e,


portanto mais ismeros estruturais, tambm chamados estereoismeros. O nmero de
ismeros dado pela expresso 2n onde n o nmero de carbonos assimtricos. Por
exemplo, em aldoexoses h 4 Cs assimtricos, logo o nmero de ismeros 24 =16, sendo 8
da forma D e 8 da forma L. Mas, as estruturas lineares como representadas na Figura 10
tanto para pentoses como para hexoses so poucos estveis em soluo, formando
estruturas cclicas segundo a reao mostrada na Figura 11. Esta uma reao bem
conhecida da qumica orgnica, pela qual um lcool (OH) faz uma adio nucleoflica a
carbonila de um aldedo, formando um composto de condensao conhecido como
semiacetal. No caso do exemplo da Figura 11 a hexose a D-glicose e, como a figura
mostra, a ciclizao leva ao aparecimento de outra isomeria estrutural devido s duas
posies possveis do OH do C1 em relao ao plano do anel, gerando os ismeros e .
importante enfatizar que o OH do C1 no quimicamente equivalente aos demais OHs que
so alcolicos, sendo por isso chamado de OH glicosdico. A existncia do OH glicosdico
permite que todos os monossardeos sejam oxidados em condies brandas pelo reagente
de Fehling, uma reao de oxido-reao na qual os OHs alcolicos no participam.

Figura 12. Nomenclatura para estereoismeros.

35

2. Conforme exemplificado na Figura 12 h uma nomenclatura especificamente designada para


distinguir pares de estereoismeros. Enantimeros possuem estruturas isomricas que so
uma imagem especular da outra, por exemplo, cada membro da famlia D de hexoses
mostrada na Figura 10 tem um, e, somente um, enantimero na famlia L. So epmeros
pares de estereoismeros que diferem apenas pela configurao de um C assimtrico. So
anmeros os dois ismeros resultantes da posio do OH glicosdico do C1 na estrutura
cclica da hexose. E, finalmente, so denominados diastereoismeros pares de ismeros que
no caem em nenhuma das categorias anteriores.
3. Ligao glicosdica: os monossacardeos podem se apresentar na forma de oligo ou
polissacardeos, onde os monmeros so ligados atravs de ligaes glicosdicas.
Oligossacardeos so formados por um pequeno nmero de monossacardeos, resultantes
da condensao de um OH glicosdico com um OH alcolico, como exemplificado abaixo
pela dimerizao de duas molculas de -glicose por ligao 1-4, originando o dissacardeo
maltose:

Caso a ligao glicosdica envolva a condensao dos dois OHs glicosdicos como o caso
da trealose, uma 1-1-diglicose, o dissacardeo no pode ser oxidado pelo reagente de
Fehling (dissacardeo no redutor). J a maltose, que possue um OH glicosdico livre um
dissacardeo redutor, sendo oxidado pelo reagente de Fehling.
4. Polissacardeos so polmeros constitudos de centenas ou milhares de resduos de
monossacardeos, geralmente glicose, formando cadeias lineares, como a celulose (1-4poliglicose), ou cadeias ramificadas, como o glicognio e o amido.
O glicognio altamente ramificado, as suas cadeias lineares so formadas por ligaes 14-glicosdicas e suas ramificaes decorrem de ligaes 1-6-glicosdicas (Figura 13). O
glicognio apresenta uma nica extremidade redutora livre (C1 no resduo final na ltima
molcula de glicose da cadeia) e inmeras extremidades no redutoras. A partir das
extremidades no redutoras h acrscimo ou retirada de resduos do polmero. Portanto, as
molculas de glicognio no tm tamanhos definidos.

36

Figura 13. Glicognio Poli (1-4) (1-6) glicose.


5. Ciclodextrinas so compostos derivados da unio de molculas de glicose por ligao 1-4,
gerando sistemas cclicos denominados em funo do nmero de unidades de glicose de ,
e CDs macrociclos estes que tem, 6, 7 e 8 unidades glicosdecas. Estes compostos so
produzidos por enzimas extra-celulares. Estes ciclos tem a cavidade interna relativamente
hidrofbicas e permitem a incluso de vrios compostos, na ordem estes so benzeno,
naftaleno e antraceno. Estes compostos tem grande aplicao em Farmcia e na indstria de
alimentos.

Exerccios 8
1) Desenhe o conjunto dos ismeros de D-aldoses de 6 C, atravs das frmulas de projeo de
Fisher. Quantos epmeros possue uma hexoaldose. Identifique todos os epmeros de D-glicose.
Existem pares enantiomricos na famlia D de monossacardeos. Explique.
2) Descreva o fenmeno da mutarrotao de D-glicose, incluindo as reaes qumicas pertinentes
com as respectivas frmulas estruturais dos reagentes. O que este fenmeno tem a ver com os
conceitos de C anomrico e anmeros.
3) Compare os dissacardeos maltose e sacarose, identificando a ligao glicosdica em cada caso.
Por que maltose redutora e sacarose no .
4) Analise a estrutura do glicognio. Procure destacar as vantagens e desvantagens da funo
deste polmero como composto de reserva energtica.

37

5) Verifique as principais caractersticas dos polissacardeos estruturais, comparando celulose,


quitina e glicosaminoglicanos (estes tambm chamados mucopolissacardeos).
6) A poro de natureza sacardica de algumas glicoprotenas pode servir como stio de
reconhecimento celular. Para desempenhar esta funo, os oligossacardeos ou glicoprotenas
devem ter a capacidade de formar um grande nmero de diferentes estruturas. Qual dos dois
pode produzir uma maior variedade de estruturas: oligopeptdeos compostos de cinco resduos
de diferentes aminocidos ou oligossacardeos compostos de cinco resduos de diferentes
monossacardeos? Explique.
7) Frutose, o principal acar do mel, comumente usada como adoante de alimento. Este acar
na forma -D-piranose provavelmente a substncia mais doce conhecida. A forma -Dfuranose muito menos doce.
a) Quais so as estruturas da -D-frutopiranose e -D-frutofuranose?
b) A doura do mel diminui ao deix-lo em repouso e ao mesmo tempo aumentando a temperatura.
Explique.
8) Interconverso das formas de D-galactose. Uma soluo recm-preparada da forma de Dgalactose (1g/ml em um tubo polarimtrico de 1 dm) mostra uma rotao ptica de + 150,7o.
Quando deixada em repouso por um longo perodo de tempo a rotao decresce gradualmente
at atingir um valor de equilbrio igual a + 80,2o. Em contraste, uma soluo recm-preparada
(1g/ml) da forma mostra rotao tica de apenas +52,8o . Quando esta soluo deixada em
repouso por vrias horas a rotao aumenta at o valor de equilbrio igual a +80,2o , valor
idntico quele observado para a -D-galactose.
a) Escreva as frmulas de projeo de Haworth das formas e da D-galactose. Qual
caracterstica distingue as duas formas?
b) Por que a rotao de uma soluo recm-preparada da forma decresce gradualmente com o
tempo? Explique por que solues das formas e (de concentraes iguais) atingem o mesmo
valor de rotao ptica no equilbrio?
c) Calcule a composio percentual das duas formas de galactose no equilbrio.
9) Esquematize as , e CDs indicando o tamanho da cavidade e a forma de cone truncado.
Como voc acha que o processo de formao de complexos de incluso? Qual a cintica dos
mesmos?

38

CIDOS GRAXOS, LPIDEOS & MEMBRANAS


1.

Lpides ou lipdeos so substncias biolgicas solveis em solventes orgnicos, como


clorofrmio e metanol e, praticamente, insolveis em gua. Os lpides compreendem: a) cidos
graxos, em geral na forma de triacilglicerois; b) glicerofosfolpides; c) esfingolpides; d) colesterol
e derivados.

2. cidos graxos so cidos carboxlicos com longas cadeias hidrocarbonadas, encontrados na


forma de tri-esteres de glicerol. A maioria possui um nmero par de C, predominando os de 16 C
(cido palmtico) e os de 18 C (cido olico). Grande parte apresenta dupla ligao (insaturado)
e muitos so poli-insaturados.
3. As propriedades fsicas dos cidos graxos dependem do grau de insaturao da cadeia
hidrocarbonada. As molculas dos cidos graxos saturados so muito flexveis, facilitando a
atrao e coeso entre si. Duplas ligaes entre C impe rigidez cadeia, tornando-a menos
flexvel e limitando a coesividade entre as molculas do cido graxo. Em conseqncia disso, a
temperatura de fuso (transio de fase slido/lquido) diminui com o grau de insaturao dos
cidos graxos.
4. Os triacilglicerdeos desempenham um papel de reserva de energia metablica. Algumas de
suas propriedades fsico-quimicas so ideais para essa funo: a) elevado grau de reduo de
seus C, maximizando a quantidade de energia livre liberada na oxidao e b) alta
hidrofobicidade, permitindo estocagem livre de gua (estoques anidros). No por acaso que os
triglicerdeos compem cerca de 90% da reserva de energia metablica e tambm da dieta
lipdica dos humanos.
5. A reserva de triacilglicerdeos do tecido adiposo mobilizada atravs da hidrlise a glicerol e
cidos graxos livres, catalisada por lpase especfica. Os cidos graxos livres so carregados
pela corrente sangunea na forma de complexos com albumina, que representa 50% da protena
do plasma.
6. Molculas anfiflicas, como lipdeos com uma nica cauda hidrofbica, cidos graxos livres e
detergentes, quando em soluo aquosa e acima de um limiar de concentrao (concentrao
micelar crtica ou cmc) formam agregados globulares chamados micelas.
7. Por outro lado, lipdeos com duas caudas hidrofbicas, como glicerofosfolipdeos e
esfingolipdeos, tendem a formar bicamadas lipdicas, que so a base estrutural das membranas
biolgicas.

Micela

Bicamada
39

8. As membranas biolgicas so compostas por protenas associadas a uma matriz de bicamada


lipdica. As protenas que compe as membranas pertencem a duas categorias: a) integrais ou

Modelo de mosaico fludo para membranas biolgicas


intrnsecas e b) perifricas ou extrnsecas. Este arranjo estrutural foi originalmente proposto em
1972 por Singer e Nicholson como o modelo de mosaico fludo para as membranas biolgicas,
que foi plenamente confirmado por resultados experimentais estruturais e funcionais.

9.

As membranas so barreiras hidrofbicas que oferecem grande resistncia passagem de


solutos hidroflicos, cuja permeao exige protenas transportadoras especficas, conforme

40

esquematizado na figura abaixo. Desta maneira a membrana, atravs de transportadores


especficos, regula o transporte de metabolitos entre compartimentos celulares.

10. Um exemplo clssico de transporte a tomada de glicose pela hemcia mediada por um
transportador especfico, cuja velocidade depende da concentrao externa de glicose e
obedece a uma curva hiperblica de saturao j bem conhecida da cintica enzimtica,
sendo Kt anlogo a Km:
Esta forma de transporte conhecida como transporte passivamente mediado ou difuso
facilitada. Trata-se de um processo exergnico, pelo qual o soluto, no caso a glicose,
atravessa espontaneamente a membrana indo do compartimento de maior para o de menor
concentrao.
11. Existem 5 transportadores conhecidos que mediam a difuso facilitada de glicose em
humanos: GLUT1 a 5, cujos Kts so diferentes para atender as necessidades funcionais dos
tecidos nos quais so expressos. GLUT1 o transportador em hemcias, j GLUT2
expresso no fgado e clulas beta do pncreas, enquanto GLUT4 aparece no msculo
esqueltico, tecido adiposo etc.
12. Mas, no epitlio do intestino a glicose obtida da dieta transportada para dentro da clula
contra a gradiente de concentrao, portanto atravs de um processo endergnico que exige
consumo de energia metablica para ocorrer e referido como transporte ativo. Neste caso o
transportador chamado simport, pelo qual a glicose transportada junto com Na+ e
termodinamicamente possvel porque existe um gradiente eletroqumico de Na+ de fora para
dentro da clula. H mltiplas formas de transporte ativo, das quais este exemplo da glicose
apenas uma delas. Grande parte da energia metablica consumida pelas clulas se deve

41

manuteno da enorme diversidade de transportadores que promovem a transferncia de


metabolitos e ons contra gradientes de concentrao.

Exerccios 9
1) O que concentrao micelar crtica. Como varia tamanho e forma de micelas formadas por
anfiflicos de uma nica cauda hidrocarbonada? Explique.
2) Uma hiptese central na pesquisa de membranas que os lipdeos da membrana devem ser
fludos (em oposio a "congelados") a fim de que a membrana possa desempenhar suas
funes. O apoio para esta hiptese fornecido pela observao de que a composio de cido
graxo das membranas pode ser alterada pelas condies nas quais a bactria cresce. Por
exemplo, se a bactria est crescendo em temperatura menor que a normal, as quantidades
observadas de cidos graxos insaturados (relativas ao contedo de cido graxo saturado) esto
acima do normal.
Contrariamente, se a bactria est crescendo em temperatura acima da normal, as quantidades
observadas de cidos graxos insaturados nos lipdeos da membrana (relativas aos cidos
graxos saturados) esto abaixo do normal.
a) Sugira razes para o fato de que o contedo lipdico na membrana bacteriana deve ser fluido
para que a membrana intacta opere apropriadamente.
b) Explique como a alterao observada nos nveis dos cidos graxos insaturados relativa aos nveis
dos cidos graxos saturados, em diferentes temperaturas de crescimento, apia a hiptese da
fluidez da membrana.
3) Fornea uma explicao termodinmica para o fato de que molculas de fosfolipdeo difundem
rapidamente no plano da bicamada, mas muito lentamente mudam de uma face oposta.
4) Descreva os mecanismos pelos quais detergentes extraem protenas integrais de membrana,
mantendo-as em soluo.
5) Explique porque soda funciona bem para desentupir pias entupidas com gordura animal.
6) Para saber se uma bactria tomava leucina e etileno glicol por transporte mediado ou no
mediado, foram feitas medidas de velocidade inicial de tomada em funo da concentrao de
ambas substncias, resultando na tabela fornecida abaixo. O que voc conclui do exame dessa
tabela? Explique e calcule Kt e Vmax se encontrar evidncias de transporte mediado.

42

Componente
Leucina

Concentrao [M]

Velocidade Inicial (unidades arbitrrias)

-6

110

-6

220

-6

480

-5

830

-5

1700

-4

2600

-4

3100

-3

3200

-3

-3

5 x 10

0,01

10

0,05

50

0,1

100

0,5

500

1,0

1000

1 x 10
2 x 10
5 x 10
1 x 10
3 x 10
1 x 10
5 x 10
1 x 10

Etileno glicol

1 x 10

7) Clulas epiteliais de intestino de camundongo isoladas em cultura transportam L-leucina e Dleucina mostrando Kt (mM) e Vmax, respectivamente iguais a: 0,24 e 420 para L-leucina e 4,7 e
310 para D-leucina, ambos em presena de Na+ no meio de cultura. Mas na ausncia de Na+ , Lleucina mostra 0,24 e 23 enquanto D-leucina mostra 4,7 e 5 para Kt (mM) e Vmax,
respectivamente. Classifique esse transportador de leucina quanto ao tipo e mecanismos de
ao. Que efeitos voc esperaria se nesse meio de cultura fosse colocada valinomicina (ionforo
de Na+)? E se fosse dissolvida ouabana (inibidor da ATPase Na+/K+) no meio de cultura?
Explique.
8) O pH e a absoro de drogas. A droga aspirina, intensamente receitada, um cido fraco com
um pKa de 3,5. A aspirina absorvida para o sangue atravs das clulas de revestimento do
estmago e do intestino delgado. Para uma substncia ser absorvida ela deve atravessar
facilmente a membrana celular. A passagem atravs da membrana celular determinada pela
polaridade da molcula: molculas inicas (carregadas) e molculas altamente polares passam
lentamente, enquanto aquelas neutras e hidrofbicas passam rapidamente. Como o pH do suco
gstrico cerca de 1 e o pH no intestino delgado, cerca de 6, pergunta-se:
a) Escreva por frmulas estruturais a ionizao reversvel da aspirina.
b) Onde a aspirina mais absorvida para a corrente sangunea, no estmago ou no intestino
delgado? Justifique claramente a sua escolha.

43

COLESTEROL E LIPOPROTENAS

1. O colesterol do organismo humano pode ser obtido atravs da dieta ou por sntese
endgena. Os principais rgos responsveis pela produo de colesterol so o fgado e o
intestino, que produzem em torno de 25 % do colesterol endgeno.
2. A sntese de colesterol ocorre no citoplasma das clulas, sendo a acetil-CoA a precursora de
todos os tomos de carbono presentes na molcula e o agente redutor, NADPH. A via
composta por vrias reaes em que a acetil-CoA forma unidades de cinco carbonos, com
estrutura similar do isopreno, que se polimerizam em um intermedirio linear que, aps,
ciclizao, origina o colesterol.
3. A sntese inicia-se com a condensao de duas molculas de acetil-CoA, produzindo
acetoacetil-CoA; esta condensa-se com outra molcula de acetil-CoA, produzindo 3-hidrxi3-metilglutaril-CoA

(HMG-CoA).

As

enzimas

que

catalisam

estas

reaes

so,

respectivamente, tiolase e hidroximetilglutaril-CoA sintase. A HMG-CoA a seguir reduzida


a mevalonato, custa de 2 NADPH, numa reao catalisada pela HMG-CoA redutase, uma
enzima ligada ao retculo endoplasmtico. Esta a reao limitante da sntese do colesterol,
sendo controlada por vrios fatores.
4. O mevalonato sofre duas fosforilaes, que consomem 3 ATP, e uma descarboxilao,
originando a unidade isoprenide, o isopentenil-pirofosfato
5. Um total de 6 molculas de isopentenil-pirofosfato so consumidas para formar esqualeno, o
ltimo intermedirio linear da via. A sntese de esqualeno processa-se atravs de reaes de
isomerizao, condensao, reduo por NADPH e eliminao de pirofosfato.
6. A etapa final consiste na ciclizao do esqualeno, incluindo consumo de O2 e NADPH,
remoo de grupos metila e migrao de ligaes duplas, que levam, finalmente, produo
de colesterol.
7. O colesterol, alm de ser um importante componente das membranas biolgicas, precursor
dos cidos biliares, hormnios esterides e vitamina D.
8. As lipoprotenas so responsveis pelo transporte de lipdeos no nosso organismo.
Consistem de uma poro protica mais externa e uma poro no-protica (nesse caso,
lipdica) mais interna. O cerne lipdico composto por lipdeos mais apolares (triglicerdeos e
steres de colesterol) e/ou mais polares (fosfolipdeos e colesterol livre). So classificadas de
acordo com sua densidade (quanto maior o cerne lipdico, menor a densidade). Em ordem
crescente de densidade, temos:

Quilomcrons: realizam o transporte preferencial de triglicerdeos (85 %) do intestino


para

fgado

tecidos

extra-hepticos

(tecidos

adiposo,

cardaco

musculoesqueltico).

VLDL (Lipoprotena de densidade muito baixa): transporta triglicerdeos (50 %)


endgenos do fgado para tecidos extra-hepticos.

44

IDL (Lipoprotena de densidade intermediria): responsvel pelo transporte de


colesterol e steres de colesterol para tecidos extra-hepticos, sendo metade captada
pelo fgado e a outra, convertida em LDL.

LDL (Lipoprotena de densidade baixa): realiza o transporte de colesterol (8 %) e


steres de colesterol (37 %) para tecidos perifricos e est envolvida na regulao da
sntese de colesterol.

HDL (Lipoprotena de densidade alta): faz o transporte reverso do colesterol, isto ,


dos tecidos extra-hepticos para o fgado, e para tecidos que o utilizam como
precursor biossinttico.

9. Conforme mencionado, a regulao da sntese do colesterol incide principalmente sobre a


reao catalisada pela HMG-CoA redutase, atravs de alteraes na atividade e
concentrao da enzima. Sua atividade controlada por fosforilao/desfosforilao: a forma
desfosforilada ativa e a fosforilada, inativa. Isto quer dizer que a adrenalina e o glucagon
inibem a enzima, enquanto a insulina a estimula. A concentrao da HMG-CoA redutase
regulada por variao da expresso gnica e da velocidade de degradao da enzima. O
colesterol e o mevalonato inibem a sntese e a traduo do RNAm da HMG-CoA redutase e
aumentam a velocidade de degradao da enzima.
10. A sntese de receptores de LDL tambm inibida por nveis elevados de colesterol
intracelular. As LDL penetram nas clulas por endocitose adsortiva, que se inicia pela ligao
da lipoprotena ao seu receptor presente na membrana plasmtica. Sendo assim, uma
diminuio do nmero de receptores de LDL propicia uma reduo no aporte de colesterol
para as clulas. A conseqncia da menor incorporao celular de LDL-colesterol o
aumento da sua concentrao no plasma.

Exerccios 10

1. Qual a principal forma de excreo do colesterol? Que importante papel fisiolgico tem a
excreo do colesterol? Explique quimicamente.

2. Quais so os principais grupos de hormnios esterides? Onde so sintetizados e quais so


seus papis fisiolgicos? Esquematize a sntese de tais hormnios a partir do colesterol.
3. Quais so as conseqncias de uma doena hereditria caracterizada pela ausncia de
receptores de LDL? Explique.
4. Por que comum referir-se ao LDL-colesterol e HDL-colesterol como colesterol mau e
bom, respectivamente?
5. Por que uma dieta com restrio em colesterol, no suficiente para reduzir seus nveis
plasmticos?

45

6. Uma das terapias medicamentosas utilizadas para a normalizao do colesterol plasmtico


o uso de resinas, como a colestiramina. Explique o processo fisiolgico sobre o qual tal
interveno atua e de que forma essas resinas funcionam.
7. As estatinas (sinvastatina, lovastatina, pravastatina, atorvastatina, etc.) so medicamentos
utilizados para reduzir os nveis plasmticos de colesterol. Baseados na estrutura qumica de
tais compostos, explique seu mecanismo de ao.

8. Por que indivduos diabticos possuem maior predisposio para nveis elevados de
colesterol? Justifique bioquimicamente.
9. Que so Sais Biliares e quais as funes dos mesmos? Quais os pigmentos que
conferem a colorao s fezes?

46

BIOENERGTICA, TERMODINMICA e ATP

1. A variao de energia livre padro diretamente relacionada constante de equilbrio:


Go = -2.3RT log Keq
2. A composio de um sistema de reao (uma mistura de reagentes e produtos) tende a uma
variao contnua at que o equilbrio alcanado. No equilbrio, as taxas de reao para um
lado e para outro so exatamente iguais. As concentraes de reagentes e produtos no
equilbrio definem a constante de equilbrio. Na reao:
A + B

C + D , a constante de equilbrio dada por:

Keq = [C][D] / [A][B]


3. Quando um sistema no est em equilbrio, ele tende ao equilbrio, e a magnitude desta
tendncia pode ser medida como a variao de energia livre da reao, G. A energia livre de
Gibbs (G), uma propriedade termodinmica, definida pela equao: G = H TS, onde H, T e S
so respectivamente entalpia, temperatura absoluta e entropia, todas tambm propriedades
termodinmicas.
4. Numa transio de estado a temperatura (T) e presso constantes (condies comuns s
reaes bioqumicas) a variao de G (G) : G = H - T
S.
Se se trata de uma reao bioqumica, H o calor de reao. Quando H positivo a reao
endotrmica, se H for negativo a reao exotrmica. Nestas condies, a espontaneidade da
reao definida pelo valor de G: se G negativo, a reao espontnea, sendo
denominada

exergnica.

Se,

ao

contrrio,

for

positivo,

reao

no

ocorre

espontaneamente e denominada endergnica. Portanto, a reao ocorre no sentido em que a


energia livre total diminui.
4. No equilbrio, G = 0. Logo, possvel demonstrar a validade das seguintes igualdades:
G = G + 2,3 RT logB/A

B/A = K

G = - 2,3 RT logK

5. Em condies padro, 25C (298K), com concentraes de reagentes e produtos iguais a 1M,
pH = 0, a variao de energia livre considerada padro, ou G. Entretanto, a maioria das
reaes bioqumicas ocorrem em pH 7,0, para as quais utiliza-se G.
6. A Figura 4 mostra esquematicamente como varia G com o desenvolvimento da reao, indicado
no eixo das abcissas como coordenada de reao

47

Energia
Livre
(G)

Estado de Transio
(T)
G*

Estado Inicial (S)

G'

Estado Final (P)

Coordenada de Reao
Figura 4. Variao de energia livre (G) no decorrer de uma reao genrica.
Para que a reao ocorra, necessariamente tem-se Gfinal < Ginicial, isto , G negativo. Um ponto
importante a ser destacado que o valor de G permite prever se a reao pode ocorrer, mas
no a velocidade com que a reao atinge o equilbrio. A velocidade de reao depende da
energia livre do Estado de Transio que maior que do que o dos reagentes no Estado Inicial,
isto , G* positivo. Quanto maior o valor de G*, menor ser a velocidade de reao.
8. Na reao genrica A B a velocidade (v) proporcional a [A], isto , v1=k1[A]. A velocidade da
reao inversa ser, consequentemente, v-1=k-1[B]. k1 e k-1 so constantes de velocidade e
reaes como AB e BA so ditas de primeira ordem, porque as suas respectivas
velocidades dependem de concentrao molar de um nico reagente elevado potncia 1. As
constantes de velocidade k1 e k-1 so diferentes da constante de equilbrio da reao, K=[B]/[A].
No estado de equilbrio, por definio, v1=v-1 e, portanto, formalmente, K=k1/k-1. As reaes
representadas pelas equaes seguintes: 2AB e A+BC so de segunda ordem, cujas
velocidades so, respectivamente, v=kA[A]2 e v=kAB[A][B]. Notar que a ordem da reao no
coincide necessariamente com a estequiometria da equao qumica.
9. As quinases formam uma classe muito importante e abundante de enzimas, que se caracterizam
por catalisar a transferncia de um grupo fosfato de alta energia para uma outra substncia
receptora.
10. So chamados compostos de alta energia substncias orgnicas com o grupo fosfato em
ligaes anidrido ou fosfoenol, cuja hidrlise libera fostato inorgnico (Pi) com um G0 negativo
e em valor absoluto superior a 8kcal/mol. Outros compostos fosforilados com o fosfato em
ligaes ester ou tioester tambm mostram um G0 de hidrlise negativo, mas de valor absoluto

48

da ordem de 3kcal/mol. Estas classes de compostos esto ilustradas na Tabela 2. O principal


composto fosforilado da clula o ATP; cuja frmula estrutural est na Figura 5. O ATP possui
fosfato em ligaes anidrido e ester, aos quais correspondem G0 de hidrlise de,
respectivamente, -8kcal/mol e -3,5kcal/mol. Todas estas reaes so, portanto, muito
voltadas para os produtos de hidrlise, sendo praticamente irreversveis. No entanto,
nenhuma destas reaes ocorre na clula a velocidade significante se no houver catlise
por uma enzima especfica, da classe das fosfatases.

NH2
C

N
HC
N
O-

O-

O-

- O- P - O- P - O- P - O- C H
2
O

Ad e n i n a

C H

C
N

O
H

HO

Ri b o s e

OH

A MP
ADP
AT P

ATP = Adenosina 5-trifosfato


Na clula:

[ATP] + [ADP] + [AMP] = Constante

FIGURA 2
Figura 5. Frmula estrutural do ATP.

49

Tabela 2. Compostos fosforilados.


R
C

R
P

H2O

CH2

CH3

Fosfoenol

cetona

Go' = - 13.000 cal/mol

Pi
cido

R
C

C OH

H2O

H2O

CoA

R OH

H2O

H2O

ADP

H2O

AMP

H2O

A OH +
lcool
(Adenosina)

Adenosina monofosfato

Pi = fosfato inorgnico = HPO 2- (pH=7,4)


4
P = PO32-

Go' = - 8.000 cal/mol

Pi
cido

cido
+

Go' = - 3.000 cal/mol

tiolcool

cido

Adenosina difosfato
AMP

cido

O
cido

Go' = - 3.000 cal/mol

Pi

R C OH + HS-CoA

Adenosina trifosfato
ADP

lcool

O
Tioster

ATP

cido

cido

ster fosfrico

R C S

Go' = - 8.000 cal/mol

Pi

O
Anidrido fosfrico

R O

Go' = - 8.000 cal/mol

Pi
cido
Pi
cido

Go' = - 3.500 cal/mol

Na clula:
[ATP] + [ADP] + [AMP] = constante

11. No metabolismo muito importante a transferncia de fosfatos de um composto fosforilado de


alta energia para outro. Uma das reaes chave deste tipo :
fosfoenolpiruvato + ADP ATP + piruvato
G0=-5kcal/mol
Como esta reao no ocorre sem catlise, seu controle pela clula feito atravs de uma
enzima quinase especfica.

50

12. Alm das quinases que catalisam a transferncia de grupo fosfato do ATP para metablitos,
existem as quinases que tem transferem grupos fosfato entre protenas. Essas enzimas so
genericamente referidas como quinases de protena ou, simplesmente, protena-quinases.
H alguns milhares de protena-quinases diferentes em um organismo, que catalisam a
transferncia de fosfato de ATP para o grupo OH da cadeia lateral de resduos especficos de
serina e treonina formando um ster de fosfato. As reaes deste tipo so genericamente
chamadas de fosforilaes e so modificaes covalentes que causam mudana de
conformao das protenas, alterando sua atividade biolgica. Por exemplo, um grande nmero
de enzimas so fosforiladas para sofrer uma transio do estado inativo ao ativo ou vice-versa.
Mais raramente as protenas so fosforiladas no grupo enlico de resduos de tirosina.

Exerccios 11
1) Defina reaes exotrmicas e endotrmicas. Qual a relao entre estes conceitos e a funo
termodinmica entalpia?
2) Defina reaes exergnicas e endergnicas. Qual a relao destes conceitos com G0.
3) G0 caracterstico de cada reao (desde que a temperatura seja constante) e no varia
com as concentraes de reagentes e produtos no equilbrio. G, por outro lado, no
caracterstico da reao, podendo assumir qualquer valor em funo das concentraes
iniciais de reagentes e produtos (quociente Q na expresso de G). Mostre por que estas
afirmaes so verdadeiras discutindo a expresso que relaciona G0 e G.
4) Na reao genrica AB Keq=103. Qual o valor de G0? No ponto de equilbrio as
concentraes molares de A e B podem variar? Como varia G com as concentraes
molares iniciais de A e B?
5) Ainda para a reao AB (questo 4) proponha uma condio na qual a reao inversa seja
espontnea. Mostre que a sua proposta possvel calculando o respectivo G. Esta questo
possui mltiplas respostas ou apenas uma resposta nica?
6) Para a reao AB (questo 4), se a constante de velocidade de primeira ordem, k1 for igual
a 10, qual deve ser o valor da constante k-1 para a reao inversa? Para um mesmo K,
constante de equilbrio, pode haver mltiplos valores de k1 e k-1 ? Qual a interpretao
termodinmica para a sua resposta?

51

7) Considerando a equao G0 = -2,3 RT log K, sendo: R = 1,98 x 10-3 kcal/mol K; T = 298K e


2,3 RT = 1,36 kcal/mol. Calcule os valores de G0 quando K varia de 105 a 10-5. Faa uma
tabela.
8) Porque a hidrlise de ATP necessita catlise enzimtica, sendo este um composto rico em
energia? Utilize-se do grfico esquemtico de variao de G (energia livre) em funo de
coordenada de reao para responder a esta questo, definindo estado de transio e
energia de ativao.

52

METABOLISMO GERAL ESTRATEGIAS

1. Os sistemas vivos para se manterem na condio de baixa entropia (alta organizao) e


ainda fora do equilbrio termodinmico necessitam retirar energia e poder redutor do
meio ambiente. Lembre-se que as molculas sintetizadas pelo organismo via de regra
esto no estado reduzido, como acares e lipdios. Lembre-se tambm que os
organismos autotrficos procedem sntese de Glicose a partir de CO2, H2O e energia na
forma de Luz (h). Este processo inclui a reduo do C (CO2) (Nox=4) para CH2O (hidrato
de Carbono) (Nox= 0).
2. Do ponto de vista metablico dois processos ocorrem. Queima de compostos para
obteno de energia e poder redutor = Catabolismo. Uso das energias (ATP) e poder
redutor para Snteses de macromolculas, movimentos, transportes contra gradientes etc.
Processos estes uphill.
3. A manuteno dos processos metablicos e do fluxo de materiais controlada por
meticuloso sistema enzimtico que permite o controle metablico seja em direo ao
catabolismo, seja em direo ao anabolismo. Este fino controle exercido em funo da
condio metablica, no qual vias so bloqueadas e outras so ativadas.
4. Reaes uphill podem ser tranformadas em downhill pelo acoplamento de reao
favorvel, em geral pela hidrlise de ATP, seja para ADP e Pi seja para AMP e PPi
(pirofosfato). Este PPi pode ser ainda hidrolizado duas de Pi e isto auxilia em deslocar o
equilbrio no sentido de produtos.
5. Como os produtos de reaes pouco favorveis so retirados para outras vias, isto pode
levar a reao complexo.
6. O ATP considerado a moeda corrente de Energia Livre. O ATP esta sempre sendo
formado e consumido. O ATP no depsito de energia. Depsitos de energia so
Lipdios, Glicognio, Creatina-Pi (msculo).
7. O creatina-Pi reserva de ~Pi no msculo.
8. NADH e FADH2 so os principais carreadores de eltrons nas oxidaes biolgicas. O
NADPH a maior fonte de eltrons para as redues biossintticas. Note que o grupo Pi
a nica diferena entre o NAD e o NADP, o quem permite o reconhecimento por
enzimas.
9. A Coenzima-A o carreador de grupos acila.
10. Abaixo se encontram figuras do catabolismo e anabolismo

53

Catabolismo

Anabolismo
54

Estratgia Metablica
1) Alvos: Produzir ATP, gerar poder Redutor, obter blocos para Snteses.
2) Funes do ATP : Fonte de Energia para Msculo
Transporte Ativo
Amplificao de Sinais (checar AMPc)
Biossntese
3) Hidrlise de ATP muda equilbrio acoplado por um fator de 108!!
4)Sntese de ATP glicose 2 ATP (vantagem a rapidez)
Ac. Graxos 30/32 ATPs (cido palmtico)
5)Via das Pentoses NADPH (biossnteses)
6)Blocos DHP glicerol TAGs
PEP aas aromticos
Acetil-CoA unidades de 2C
Succinil-CoA porfirinas
Pentose Ribose 5 Pi
7)Degradao Sntese, porm em geral exergnicos graas ao ATP.
8)Atividade Enzimtica Chave para Sucesso
9) Controle do Metabolismo em geral no Passo Irreversvel
Transforma processos de msegundos para Segundos
10)Modificao Covalente
11)Nvel de Enzimas (controle da transcrio). Sntese vs. Degradao Hormnios
12)Compartimentalizao de Processos. Ex. mitocndria, fluxo de metablitos.

Exerccios 12

1) Qual o valor do G0 para hidrlise do ATP? E do PPi?


2) Escreva a forma do ATP e esquematize quais grupos podem ser denominados ~Pi?
3) Por que o ATP rico em energia livre? Qual as formas do ATP no pH fisiolgico? Por que
o ATP em geral esta acompanhado do on Mg2+?
4) Qual o fator que a hidrlise de ATP desloca equilbrios?
5) Mostre as estruturas do NAD, do FAD e do NADP oxidados e reduzidos. Saliente quais
grupos das molculas so oxidados e reduzidos.
6) Como a estrutura da Co-Enzima A?
7) Quais as vitaminas presentes no NAD, FAD e CoA.
55

8) O que significa Carga Energtica e Potencial de fosforilao?


9) Discuta os 12 tens apresentado no tema acima Estratgia Metablica.

56

GLICLISE

1. A gliclise a principal via catablica da glicose compreendendo as 10 reaes enzimaticamente


catalisadas que so mostradas na figura abaixo e cuja estequiometria total pode ser observada
na equao qumica seguinte:
Glicose + 2 NAD+ + 2 ADP + 2 Pi 2 Piruvato + 2 NADH + 2 ATP + 2 H2O + 2 H+
A gliclise, como todas as vias catablicas, exergnica e a equao acima corresponde a
Go = -43,4 kJ/mol. Mas o dado da variao de energia livre mais interessante em termos de
G, cujo valor exato depende de cada clula especfica, por exemplo, em msculo cardaco
estima-se que seja igual a 74,0 kJ/mol.

2. A finalidade da gliclise obteno de energia, como a equao estequiomtrica indica, cada


molcula de glicose degradada a duas de piruvato e parte da energia livre liberada nesta
degradao retida nos produtos na forma de 2 NADH e 2 ATP.

3. A reao que permite a obteno de NADH a nica de oxido-reduo da gliclise, pela qual
gliceraldedo-3-P oxidado a glicerato-1,3-bisP, atravs da ao oxidante de NAD+ catalisada
pela enzima gliceraldedo desidrogenase. A manuteno da capacidade oxidante da gliclise
exige que NADH seja re-oxidada a NAD+ , uma alternativa para isso apresentada na figura,
atravs da reao pela qual NADH reduz piruvato a lactato, recuperando NAD+. Esta alternativa
ocorre no msculo esqueltico com baixos nveis de O2.

4. J ATP produzido em duas reaes distintas pelas quais um radical fosforil transferido de,
respectivamente, glicerato-1,3-P e P-enolpiruvato para ADP, em transferncias catalisadas por
glicerato-1,3-P-quinase e P-enolpiruvato-quinase. Esta maneira de fosforilao de ADP
conhecida como fosforilao a nvel do substrato, para distingui-la da fosforilao oxidativa da
mitocndria que ser vista mais adiante.

5. Na gliclise, h 3 reaes de fosforilao irreversveis catalisadas, respectivamente, pela


hexoquinase, fosfofrutoquinase e piruvato-quinase, que funcionam como marca-passos da via,
cuja regulao se d por um elaborado sistema de controle alostrico das enzimas.

6. Diversas outras hexoses, como frutose, galactose e manose, tambm so metabolisadas pela via
glicoltica.

7. A gliclise em condies anaerbicas tem energtica e funes variadas, conforme o organismo.


Cabe fazer dois destaques importantes.

8. Em vertebrados, encontram-se msculos esquelticos muito pobres em mitocndria, que so


especializados para produzir ATP a partir de gliclise anaerbica, cuja energtica obedece a
seguinte reao geral: Glicose 2Lactato + 2H+ ; Go = -196kJ/mol. Mas, parte dessa energia
livre liberada que seria dissipada (61kJ/mol) retida na forma de 2ATP produzidos por mol de
glicose degradada. Deve-se ainda enfatizar que o lactato no descartado, pois vai ser

57

aproveitado no fgado, aonde reoxidado a piruvato, alternativa metablica importante a ser


examinada mais frente.

9. Leveduras mostram um exemplo de gliclise anaerbica na forma da fermentao alcolica,


segundo a reao geral: Glicose 2Etanol + CO 2; Go = -235kJ/mol. Aqui tambm parte da
energia livre, +61kJ/mol, mantida com a produo de 2ATP. A parte final da fermentao
alcolica compreende duas reaes: a primeira envolve a descarboxilao de piruvato e
liberao de acetaldedo, catalisada pela enzima piruvato-carboxilase, que no existe em
animais. Na segunda reao a desidrogenase alcolica catalisa a reduo do acetaldedo por
NADH.

58

HOCH2
O

GLICLISE

OH
HO

OH

Glicose

OH
ATP

ATP

ADP

ADP

P -OCH2
O
OH

Glicose 6-fosfato

HO

OH
OH

P -O-CH2 O CH2OH
HO
OH

Frutose 6-fosfato

HO
ATP

ATP

ADP
P -O-CH2 O

ADP

CH2O- P

HO OH

Frutose 1,6 bisfosfato

HO

HC=O

H2C-O- P
C=O

Diidroxiacetona
fosfato

HC-OH

H2C-OH

Gliceraldedo
3-fosfato

H2C-O- P
Pi
NAD+

Pi
NAD+

NADH

NADH

O=C-O- P
HC-OH

1,3 Bisfosfoglicerato

H2C-O- P
ADP

ADP

ATP

ATP

O=C-OHC-OH

3-Fosfoglicerato

H2C-O- P

COOHC-O- P

2-Fosfoglicerato

H2C-OH

H2O
COOC-O- P

Fosfoenolpiruvato

CH2
ADP

ADP

ATP

C=O

HC-OH
CH3

ATP

COO-

COO-

NAD+

NADH

CH3

Lactato

Piruvato
NAD+

NADH

59

Exerccios 13
1) Classifique as reaes da gliclise, destacando as que so de xido-reduo.
2) Equacione a reao de oxidao de gliceraldedo-3-fosfato, destacando o oxidante e o redutor.
3) Na reao do item 2) parte da energia utilizada para produzir ATP. Mostre como isso possvel,
equacionando as etapas relevantes da reao. Defina fosforilao ao nvel do substrato.
4) Equacione a reao lquida da transformao de glicose em piruvato. Como regenerada a
capacidade oxidante do sistema NAD+/NADH necessria atividade glicoltica nos glbulos
vermelhos humanos (que no tm mitocndria) e na cultura de levedo sem O2 (fermentao).
5) Examine uma tabela com as 10 reaes da via glicoltica que contenha, respectivamente, o G0
e o G das reaes. Quais so as reaes irreversveis da gliclise?
6) Porque os valores de G0 e de G da mesma reao podem ser diferentes? Para decidir se a via
glicoltica numa determinada clula reversvel ou irreversvel, que valor mais relevante, G0
ou G?
7) Uma pessoa incapaz de executar exerccios fsicos intensos e prolongados teve suas enzimas
analisadas. Todas as enzimas da via glicoltica estavam em concentrao normal, com excesso
da fosfoglicerato mutase muscular.
a) Como ser afetada a produo de energia metablica em uma clula que apresenta baixos nveis
desta enzima?
b) Como ser afetada a produo de Lactato na ausncia desta enzima? [Referncia: Di Mauro, S.;
Miranda, A.F.; Kahn, S.e Gitlin, K. - Human muscle phosphoglycerate mutase deficiency Science
1981, vol. 212, 1277-1279.
8) Calcular a porcentagem de energia armazenada pela clula ao degradar glicose pela via
glicoltica. Sabe-se que:

Glicose 2 lactato

G ' = - 47.000 cal/mol

60

CICLO DE KREBS

1. Em condies aerbicas, o destino do piruvato produzido na gliclise sofrer uma


descarboxilao oxidativa catalisada pela piruvato desidrogenase, que um complexo
multienzimtico existente no interior da mitocndria de eucariotos. Portanto, o piruvato precisa
entrar na mitocndria para ser degradado por essa via. A reao geral a seguinte:
Piruvato + CoA + NAD+ Acetil-CoA + NADH + CO2

2. O acetilCoA resultante da metabolizao do piruvato totalmente oxidado no ciclo do cido


ctrico, tambm chamado ciclo de Krebs, conforme a seguinte reao geral:
Acetil-CoA + 3 NAD+ + FAD + GDP + Pi + 2 H2O 2 CO2 + 3 NADH + FADH2 + GTP + CoA + 2 H+
O ciclo de Krebs, esquematizado na figura, compreende 8 reaes, envolvendo 8 enzimas e 8
cidos carboxlicos, di e tri-cidos, todos dispersos na matriz da mitocondria. Portanto,
comeando no piruvato e passando pelo acetilCoA, ocorre oxidao completa desses metabolitos
liberando 3CO2 sem participao de O2 molecular. Os agentes oxidantes em todas as reaes
so NAD+ ou FAD e as formas reduzidas destas co-enzimas (NADH + FADH2 ), resultantes do
processo, s so reoxidadas na cadeia respiratria, uma via especializada que se localiza na
membrana mitocondrial interna e ser considerada mais adiante.
3. O ciclo de Krebs, conforme sua reao geral indica, essencialmente catablico, pois promove
a oxidao do radical acetil a 2CO2 e retm parte da energia livre desta reao na forma de
coenzimas reduzidos que, posteriormente, serviro produo de ATP atravs da fosforilao
oxidativa. Para cumprir esta funo basta que os 8 intermedirios do ciclo ocorram em
concentraes catalticas. Mas, o ciclo possui outra funo, alm da catablica, diversos de seus
intermedirios alimentam as vias de sntese de aminocidos, lipdeos e glicose, isto , o ciclo
tem tambm funo anablica e, portanto, deve ser classificado como anfiblico. Para que o
ciclo desempenhe concomitantemente ambas as funes, catablica e anablica, as
concentraes dos intermedirios so mantidas e controladas atravs de um complexo sistema
de reaes auxiliares, conhecidas como reaes anaplerticas. Um exemplo de reao
anaplertica a carboxilao de piruvato para obter oxalacetato, catalisada pela enzima piruvato
carboxilase.
4. A transformao de piruvato em acetil-CoA, uma reao para a qual convergem diversas vias
catablicas e anablicas, alm da gliclise. Por esse motivo a piruvato desidrogenase est
sujeita a um controle altamente elaborado, compreendendo dois nveis de regulao: a) controle
alostrico atravs da inibio pelo produto, exercido por NADH e acetil-CoA; b) modificao
covalente reversvel da subunidade E1 da enzima, por fosforilao/desfosforilao.
5. As enzimas citrato sintase, isocitrato desidrogenase e -cetoglutarato desidrogenase so as
reguladoras do fluxo metablico atravs do ciclo de Krebs e esto sujeitas a controle alostrico,
envolvendo NADH como inibidor e Ca+ e ADP como ativadores.

61

Piruvato

CoASH + NAD+
CO2 + NADH

Acetil-CoA
H 2O

Oxaloacetato

CoASH

Citrato

NADH + H+

H2O

NAD+

cis-Aconitato

L-Malato

H 2O

H2 O

Fumarato

Isocitrato

Ciclo de Krebs

FADH2

NAD+

FAD

NADH + H+

Succinato

Oxalosuccinato

GTP
CO2

GDP + Pi

Succinil-CoA
NADH
+ H+

CoASH

CO2

a-Cetoglutarato
NAD+

Exerccios 14
1) Escrever a reao de formao de acetil-CoA a partir de piruvato e indicar:
a) as 5 coenzimas necessrias
b) as vitaminas envolvidas
c) a sua localizao celular
2) Como a equao qumica, estequiometricamente equilibrada, que representa a oxidao de
acetil-CoA no ciclo de Krebs? Como se pode medir o rendimento do ciclo de Krebs em termos
de coenzimas reduzidos (poder redutor) e ATP (ligaes de fosfato de alta energia).
3) Identifique os tipos de reaes que ocorrem no ciclo de Krebs, mostrando as respectivas
equaes qumicas.

62

4) Equacione a descarboxilao oxidativa de -cetoglutarato a succinato, respeitando a


estequiometria da reao. Mostre as etapas que compem esta reao com as respectivas
enzimas e coenzimas.
5) Quais so as enzimas do ciclo de Krebs sujeitas a regulao? Explique como cada uma delas
regulada.
6) Explique porque piruvato estequiometricamente convertido a CO2 na respirao de fatias de
msculo mantidas em soluo fisiolgica, enquanto oxalacetato e citrato tem efeito cataltico
neste mesmo processo. Mostre porque a respirao pode ser sustentada pelo consumo
estequiomtrico de citrato, mas no de acetato, quando o ciclo de Krebs inibido por malonato.
7) Dispondo das enzimas necessrias, a adio de que compostos far aumentar a concentrao de
oxaloacetato em um sistema in vitro que contm mitocndrias: acetil-CoA, piruvato, glutamato,
citrato ou cidos graxos?
8) Uma suspenso de mitocndrias, suplementada com acetil-CoA marcada com C14, produz CO2
marcado apenas quando suprida de oxignio. Em condies anaerbias, a adio de azul de
metileno restaura a produo de CO2 marcado, observando-se tambm a descolorao do corante
(azul de metileno reduzido incolor). Explique estes dados.

63

CADEIA RESPIRATRIA E FOSFORILAO OXIDATIVA

1. Fosforilao oxidativa o processo bioqumico pelo qual a oxidao de NADH e FADH2

produzidos na gliclise e ciclo de Krebs, ocorre acoplada produo de ATP, a partir de ADP +
Pi. Este processo se d na cadeia respiratria ou cadeia de transporte de eltrons, que
compreende um conjunto ordenado de enzimas e transportadores de eltrons inseridos na
membrana interna da mitocndria.

2. A cadeia respiratria contem 4 complexos, I,II, III e IV, ordenados por ordem crescente de
potencial redox, indo do potencial padro de NAD+/NADH (E0= -0,315V) ao do O2/H2O (E0=
+0,815V). Os eltrons so transferidos do complexo I ou II para o complexo III pela coenzima
Q (ou ubiquinona), e do complexo III para o complexo IV pelo citocromo C para chegar ao O2.
NADH e FADH2

cedem eltrons, respectivamente, aos complexo I e II. A transferncia

exergnica de eltrons do nvel redox de NADH para o de O2 (E0= 1,130V) envolve uma
diferena de energia livre liberada (G0= -218kJ/mol) que em parte retida pelo transporte de H+
do lado interno para o externo da membrana, criando o gradiente eletroqumico de prtons que
permitir empurrar o processo endergnico de fosforilao de ADP por Pi para gerar ATP,
atravs da bomba de prtons que constitui a ATP sintase (tambm conhecida com F1F0ATPase).
Espao
Intermembranar

2 H+

4 H+

2 H+
Cit C

Complexo I

Matrix
Mitocondrial

Complexo III

Complexo IV

NAD+
1/2 O2 + 2H+

NADH + H+

H2O

Complexo II
FADH2

3. A ATP sintase distinta e fisicamente separada da cadeia de transporte de eltrons. A


transferncia de 2e de NADH at O2 envolve um G0= -218kJ/mol, que gera um incremento no
gradiente de prtons suficiente para mover a ATP sintase, permitindo a produo de 3 moles de
ATP (G0= +30,5kJ/mol). Nestas condies, a ATP sintase trabalha com uma eficincia
termodinmica igual a 42%. , no entanto, necessrio destacar que quando os 2e saem do nvel

64

redox de FADH2

formam-se apenas 2ATP. Naturalmente, para uma melhor medida da real

eficincia termodinmica da fosforilao oxidativa seria preciso estimar o G da transferncia de


eltrons em vez do G0.

4. A grande quantidade de energia livre que seria dissipada na oxidao completa da glicose a CO2
e H2O [C6H12O6 + 6 O2 6 CO2 + 6 H2O; G0= -2823 kJ/mol] aproveitada para produo de
ATP, graas quase exclusivamente ao processo de fosforilao oxidativa, rendendo 38ATP por
mol de glicose (incluindo neste total 2ATP da gliclise e 2 do ciclo de Krebs).

5. Vrios mecanismos da cadeia de transporte de eltrons e de seu acoplamento sntese de ATP


foram elucidados atravs da utilizao de inibidores e desacopladores, entre os quais esto:
rotenona, amital, antimicina A, cianeto e DNP.
Rotenona e amital inibem a reduo dos complexo I e III por NADH.
Antimicina A inibe o transporte de eltrons no complexo II.
Cianeto inibe o transporte no complexo IV.
DNP desacoplador, pois promove o vazamento de H+ ,levando dissipao do gradiente
de prtons e contnuo transporte de eltrons, desacoplado da sntese de ATP.

6. A sntese de ATP a partir de ADP e Pi na mitocndria, que catalisada pela ATP sintase,
dirigida pelo processo de transporte de eltrons. Mas como a ATP sintase fisicamente separada
das protenas do transporte de eltrons, a energia livre liberada no transporte de eltrons deve
ser conservada em uma forma que possa ser utilizada pela ATP sintase. A energia livre do
transporte de eltrons conservada pelo bombeamento de H+ da matriz mitocondrial para o
espao intermembranar, criando um gradiente de H+. A volta dos prtons ao interior da
mitocndria termodinamicamente favorvel. A membrana interna da mitocndria impermevel
a prtons em toda sua extenso, exceto na ATP sintase; e ento por este canal que os prtons
atravessam a membrana, de volta matriz mitocondrial. A variao de energia livre associada ao
transporte de um prton atravs da membrana interna da mitocndria pode ser determinada
atravs de medidas da diferena de pH e do potencial de membrana estabelecidos em
mitocndrias consumindo oxignio.

Exerccios 15
1) Definir potencial de xido-reduo (E), potencial de xido-reduo padro (Eo) e potencial de
xido-reduo padro bioqumico (Eo).
2) Entre os transportadores universais de eltrons da cadeia respiratria esto NAD+ e os
nucleotdeos de flavina (FAD e FMN), quais so as diferenas entre estes transportadores de
eltrons quanto a potencial redox e forma de interao com as enzimas com as quais atuam?

65

3) Classifique os seguintes inibidores quanto a seus mecanismos de ao na cadeia respiratria: a)


rotenona; b) antimicina A; c) oligomicina e d) DNP (2,4-dinitrofenol).
4) Descreva o mecanismo de ao do DNP (2,4-dinitrofenol), mostrando porque o mecanismo de
ao deste inibidor uma demonstrao experimental importante da hiptese quimiosmtica da
fosforilao oxidativa.
5) Porque F1 e Fo so ambos necessrios para a sntese de ATP?
6) Em mitocndrias isoladas, o transporte de eltrons no ocorre na ausncia de ADP e Pi, mesmo
que haja abundncia de succinato para fornecer eltrons. Como se explica que mitocndrias nessas
condies passam a transportar eltrons e consumir oxignio se forem tratadas com DNP?
7) A relao entre energia livre padro de uma reao e o potencial redox :
i.
ii.
iii.
iv.
v.
vi.

Go = -nFE0onde n o nmero de eltrons transferidos


F a constante de Faraday (F = 23.60 cal V-1)
Eo' o, diferena de potencial padro da dupla redox.
Lactato + NAD+Piruvato + NADH + H+
Eo' (NAD+/NADH) = -0,32 V
Eo' (Piruvato/Lactato) = -0,19 V

8) A uma soluo 1 M de NAD+, NADH, Piruvato e Lactato, adicionou-se lactato desidrogenase. A)


Em que sentido a reao ocorrer? B) medida que a reao ocorre, como variam esses
potenciais redox?

9) Na hiptese do acoplamento quimiosmtico, a energia que comea na forma de potencial


qumico de reduo/oxidao, convertida na forma de potencial prton-motriz e finalmente
convertida na forma de potencial qumico de ATP. Qual a diferena entre o potencial
qumico (G) e o potencial eltrico () de um soluto distribudo dos dois lados de uma
membrana? Defina fora prton-motriz

66

CICLO DAS PENTOSES


1. Muitas funes celulares que envolvem reaes endergnicas so efetuadas graas hidrlise
exergnica de ATP. Outras reaes endergnicas, como a sntese de cidos graxos e colesterol
e a fotossntese, requerem NADPH, que tem um grande poder redutor.

2. O grupo fosforila no carbono 2 de uma das unidades de ribose do NADPH o diferencia de NADH.
NADH oxidado pela cadeia respiratria para gerar ATP, enquanto que NADPH serve como um
doador de eltrons em reaes biossintticas redutoras.

3. Na via das pentoses, NADPH gerado quando a glicose-6-fosfato oxidada a ribose-5-fosfato,


que um acar de 5 carbonos, componente de vrios compostos importantes, como ATP, CoA,
NAD+, FAD, RNA e DNA.

4. A via das pentoses tambm catalisa a interconverso de acares de 3, 4, 5, 6 e 7 carbonos, em


uma srie de reaes no oxidativas que ocorrem no citosol.

5. As reaes da via das pentoses so as seguintes:


glicose-6-fosfato desidrogenado e convertido a ribulose-5-fosfato, em trs reaes,
produzindo 2 NADPH + H+.
ribulose-5-fosfato isomerizada a ribose-5-fosfato.
Nestas reaes, 2 NADPH + H+ e uma ribose-5-fosfato so gerados para cada glicose-6-fosfato
oxidada.
ribose-5-fosfato convertida a gliceraldedo-3-fosfato e frutose-6-fosfato pela transcetolase e
transaldolase. A transcetolase catalisa a transferncia de unidades de C2 de uma cetose para
uma aldose. A transaldolase transfere unidades de C3 de uma aldose para uma cetose.
As reaes de transcetolase e transaldolase criam uma ligao reversvel entre a via das
pentoses e a via glicoltica. O resultado dessas reaes a formao de 2 hexoses e 1 triose a
partir de 3 pentoses:
C5 + C5

C3 + C7

Transcetolase

C7 + C3

C4 + C6

Transaldolase

C5 + C4

C3 + C6

Transcetolase

6. O excesso de ribose-5-fosfato formado pela vias das pentoses pode ser completamente
convertido em intermedirios da via glicoltica.

7. A primeira reao da via das pentoses, a desidrogenao da glicose-6-fosfato, praticamente


irreversvel. E essa a reao em que a via das pentoses controlada. O fator regulatrio mais
importante o nvel de NADP+, o receptor de eltrons na oxidao da glicose-6-fosfato a 6fosfogluconolactona. Alm disso, NADPH compete com NADP+ pela ligao enzima. A parte
no oxidativa da via das pentoses controlada principalmente pela disponibilidade de substratos.

8. A via percorrida pela glicose-6-fosfato depende da necessidade celular de NADPH + H+, ribose-5fosfato e ATP:

67

Quando muito mais ribose-5-fosfato requerida que NADPH + H+, a maior parte de
glicose-6-fosfato convertida a frutose-6-fostato e gliceraldedo-3-fosfato pela via
glicoltica, a transaldolase e a transcetolase convertem esses em ribose-3-fosfato.
Quando a necessidade de NADPH + H+ e ribose-5-fosfato esto balanceadas, a reao
predominante a formao de 2 NADPH e uma ribose-5-fosfato de glicose-6-fosfato pela
fase oxidativa da via das pentoses.
Quando muito mais NADPH + H+ requerido que ribose-5-fosfato, a glicose-6-fosfato
completamente oxidada a CO2, ou convertida a piruvato.

Exerccios 16
1) Mostre a parte oxidativa do ciclo das pentoses com as equaes das reaes envolvidas,
indicando os agentes oxidantes e a origem do carbono presente no CO2 liberado.
2) Compare NADH e NADPH, indicando suas funes no metabolismo de carboidratos. Explique
porque a via da pentose-fosfato muito mais ativa no tecido adiposo que no msculo.
3) Quando h necessidade de NADPH, o ciclo das pentoses pode funcionar dando um resultado
lquido que equivale oxidao total de glicose a CO2. Explique este processo atravs das
respectivas reaes estequiometricamente equilibradas.
4) Ribose 5-fosfato pode ser obtida para a sntese de nucleotdeos atravs da via das pentoses, com
ou sem oxidao da glicose. Mostre como isso possvel com as respectivas equaes
qumicas.
5) Explique como a via da pentosefosfato controlada, tendo em vista que grande parte das
reaes dessa via reversvel.
6) Compare as reaes catalisadas por transaldolases e transcetolases, indicando reagentes,
produtos e a natureza das reaes.

68

GLICONEOGNESE
1. O fgado humano precisa manter nveis mnimos da glicose circulante, porque crebro e
hemcias dependem quase exclusivamente de glicose para produo de energia. No entanto, a
reserva de glicognio heptico no suficiente para essa finalidade. Por isso, o fgado sintetiza
glicose de novo a partir de lactato, piruvato, glicerol, intermedirios do ciclo de Krebs e
aminocidos, atravs de uma via anablica chamada de gliconeognese. No jejum, mesmo o
jejum de poucas horas, a gliconeognese a principal fonte da glicose liberada pelo fgado na
circulao.
2. A gliclise, como j foi visto, um a via catablica com a finalidade de produzir energia na forma
de 2 NADH + 2 ATP a partir da degradao de glicose a piruvato de acordo com a equao
qumica seguinte:
Glicose + 2 NAD+ + 2 ADP + 2 Pi 2 Piruvato + 2 NADH + 2 ATP + 2 H2O + 2 H+
A gliconeognese tem a finalidade de sintetizar glicose a partir de piruvato, isto , faz o caminho
metablico inverso ao da gliclise. Mas, a gliconeognese, contrariamente gliclise, muito
endergnica. Para produzir glicose a partir de piruvato necessitam-se 2 NADH + 4 ATP +2 GTP,
conforme a estequiometria indicada na equao abaixo:
2 Piruvato + 2 NADH + 4 ATP + 2 GTP + 2 H2O Glicose + 2 NAD+ + 4 ADP + 2 GDP + 6 Pi + 2 H+
3. A gliconeognese utiliza enzimas glicolticas reversivelmente, mas trs dessas enzimas, a
hexoquinase, a fosfofrutoquinase e a piruvato quinase, catalisam reaes com G- muito
negativo, sendo essencialmente irreversveis. Estas reaes so substitudas na gliconeognese
por reaes exergnicas, tornando termodinamicamente favorvel a sntese de glicose a partir
de piruvato. Destas reaes, as duas primeiras correspondentes s enzimas hexoquinase e
fosfofrutoquinase, so substitudas por reaes simples de hidrlise de ligao fosfo-ster,
catalisadas, respectivamente, pelas enzimas glicose-6-P-fosfatase e frutose-1,6-bis-fosfatase. J
a terceira reao, que permite a volta de piruvato para P-enolpiruvato mais complexa e se d
em duas etapas catalisadas, respectivamente, por piruvato-carboxilase e P-enolpiruvatocarboxiquinase.
4. O balanceamento entre gliclise e gliconeognese coordenadamente controlado por um
complexo sistema de regulao enzimtica, envolvendo interaes alostricas e modificaes
covalentes. Todo esse controle est concentrado nas 3 reaes nas quais gliclise e
gliconeognese seguem reaes independentes, irreversveis e opostas, que so: 1) glicose /
glicose-6-P; 2) frutose-6-P / frutose-1,6-bisP; 3) P-enolpiruvato / piruvato.

69

Exerccios 17
1) Explique como a hemcia mantm glicose a 5 mM e G6P a 0,0083 mM, se a converso de
glicose em G6P muito exergnica. Como seriam afetadas as concentraes relativas dos
intermedirios da gliclise se a glucoquinase (Km = 5mM) fosse colocada artificialmente na
hemcia em lugar da hexoquinase (Km = 0,1mM)?
2) Na gliconeognese, como so revertidas as reaes de glicose G6P e F6P F-1,6-BP, que
so altamente exergnicas. Conceitue ciclo ftil.
3) A reverso da reao de PEP + ADP piruvato + ATP no pode ocorrer por um processo
relativamente fcil como a reverso de glicose + ATP G6P + ADP. Qual a soluo
bioqumica que os sistemas biolgicos utilizam para ir de piruvato a PEP?
4) Qual o consumo de energia na sntese de glicose a partir de piruvato, medido em equivalentes
de ATP. Indique as reaes onde h consumo. Compare o rendimento da via glicoltica com o
consumo da gliconeognese, so iguais ou diferentes?
5) Um procedimento comum para determinao da efetividade de compostos como precursores da
glicose colocar um animal em jejum at que os estoques de glicognio do fgado sejam
depletados e ento administrar o substrato em questo. Um substrato que leva a um aumento
lquido no glicognio heptico chamado de glicognico pois ele deve primeiro ser convertido em
glicose-6-fosfato. Mostre por meio de reaes enzimticas conhecidas quais das seguintes
substncias so glicognicas:
a) succinato,
b) glicerol,
c) acetil CoA,
d) piruvato e
e) butirato.
Escreva as frmulas estruturais das substncias relacionadas de (a) a (e).
6) Citar os efetuadores alostricos positivos e negativos de fosfofrutoquinase e frutose 1,6
bisfosfatase no fgado. Quais so as consequncias destes efetuadores no fluxo relativo dos
metabolitos atravs da neoglicognese e da gliclise?

70

7) O nvel de frutose 2,6 bisfosfato nos hepatcitos varia com a disponibilidade da glicose: baixo
no jejum e alto aps as refeies. Como isso se explica em termos das reaes catalisadas por
fosfofrutoquinase-2 e frutose 2,6 bisfosfatase.

71

METABOLISMO DO GLICOGNIO
1. O glicognio um polissacardeo que funciona como forma de reserva de energia em animais e
microrganismos. Em animais, o glicognio est depositado no fgado, um rgo central de
reserva de energia, e, tambm, nos msculos, onde degradado localmente. O glicognio
heptico exportado para manter a glicemia.
2. A natureza polimrica e semi-solvel do glicognio constitui-se numa maneira perfeita de
armazenar energia na forma de glicose. O estoque de glicognio do fgado na forma de glicose
causaria tamanha presso osmtica, que a viabilidade do hepatcito seria impossvel.
3. O glicognio um polmero de -D-glico-piranose altamente ramificado. Na cadeia os
monmeros so interligados por ligaes glicosdicas (14); nos pontos de ramificao a
ligao tambm glicosdica, mas (16).
4. A glicose, na forma de glicose-1-P, liberada da reserva de glicognio pela fosforlise da
ligao (14) da extremidade no redutora do polmero. Esta reao catalisada pela
glicognio fosforilase.
5. A glicognio fosforilase degrada at restarem 4 resduos antes de uma ramificao at que a
enzima desramificadora transfere 3 dos 4 resduos para outra extremidade da cadeia de
glicognio formando uma nova ligao (14). O resduo restante est ligado a cadeia pela
ligao (16) que hidrolisada pela enzima desramificadora atravs de sua atividade
(16) glicosidase.
6. Glicose-1-fosfato convertida a glicose-6-fosfato pela fosfoglicomutase, esta pode ser liberada
pela circulao no fgado pela ao da glicose-6-fosfatase ou degradada pelo msculo.
7. A sntese do glicognio se d atravs de via uma diferente da de degradao. A glicose-1-P
primeiro ativada uridina difosfato-glicose, ou simplesmente UDP-G. UDP-G o substrato da
glicognio sintase que catalisa a adio de um resduo de glicose ao carbono 4 da glicose de
uma extremidade no redutora do glicognio, liberando ainda como produto UDP. Esta reao
necessita de cadeias glicognicas pr-existentes que funcionam como PRIMER da reao,
oferecendo extremidades no redutoras para reagir com UDP-G.
Gli 6-P

Gli 1-P

Gli 1-P + UTP UDP-Gli + PPI (1-fosfato uridil transferase)


UDP-Gli + glicognio (n) UDP + glicognio (n + 1)
O UDP convertido a UTP custa da utilizao de ATP:
PPI + H2O 2 PI (pirofosfatase)
UDP + ATP UTP + ATP (nucleosdeo difosfato quinase)

72

8. A glicognio fosforilase e a glicognio sintase formam um ciclo que, respectivamente, libera e


deposita glicose-1-P no estoque da glicognio:
UTP + H2O
Glicose-1P

2 Pi
UDPG

Glicognion
Fosforilase
Pi

SintaseI
Glicognion+1

UDP

fcil notar que se estas enzimas funcionarem concomitantemente o ciclo ser FTIL, cujo
nico resultado lquido ser dissipao de energia atravs da reao:
UTP + H2O UDP + Pi
Conclui-se que, necessariamente, no hepatcito estas enzimas so coordenadamente
reguladas, isto , quando a fosforilase ativada para mobilizar glicose-1-P, a sintase
desativada, e vice-versa, conforme a necessidade celular.
9. Ambas fosforilase e sintase so reguladas por fosforilao (modificao covalente) em resduos
especficos de serina, reaes catalisadas pela mesma protena-quinase que possui dupla
especificidade, sendo por isso chamada de sintase-fosforilase quinase. A fosforilase e a sintase
so espcies fosforiladas, portanto a fosforilao, catalisada pela sintase-fosforilase quinase,
causa ativao da fosforilase e inativao da sintase.
10. A fosforilase a e a sintase I (formas ativas), por um lado, e a fosforilase b e a sintase D (formas
no ativas), por outro, so, respectivamente interconversveis. Para tanto necessrio que
fosforilase a e a sintase I sejam desfosforiladas, atravs de uma reao que requer catlise. A
principal enzima, catalisadora comum destas desfosforilaes, a fosfoprotena fosfatase 1.
11. A integrao metablica requerida pelo bom funcionamento do organismo faz com que as
interconverses coordenadas da fosforilase e sintase do glicognio no fgado, por fosforilao,
sejam controladas extracelularmente por hormnios especficos, principalmente: adrenalina,
glucagon e insulina.
12. As formas inativas fosforilase b e sintase D so intracelularmente estimuladas por fatores
alostricos

positivos,

por

razes

de

economia

interna

do

metabolismo

celular,

independentemente de controle hormonal. So estimuladores alostricos da fosforilase b e


sintase D, respectivamente, 5-AMP e glicose-6P.
Exerccios 18
1) A ativao de glicose-1-fosfato (G1-P), requer a clivagem de uma ligao de alta energia,
liberando PPi. Considere os passos de ativao de G1-P e a reao de regenerao de UTP e
analise o gasto de energia necessrio para a sntese de glicognio. Compare o gasto de energia

73

para a adio de um resduo de glicose ao glicognio, com a obteno de energia a partir da


liberao de glicose na degradao do glicognio.
2) Qual a finalidade das reservas de glicognio do fgado e msculo? Qual a principal diferena
bioqumica entre esses tecidos?
3) Equacione as etapas de mobilizao da glicose a partir do glicognio no fgado e no msculo.
Mostre:
a) no fgado (desde a fosforlise at a liberao de glicose no plasma);
b) no msculo (desde a fosforlise at o aproveitamento na gliclise).
4) Qual a funo da hidrlise de PPi no controle da sntese de glicognio?
5) Esquematize as etapas de degradao total do glicognio, indicando as enzimas envolvidas,
reagentes e produtos da reao.
6) Mostre como so coordenadas sntese e degradao do glicognio. Restrinja-se ativao e
inativao da fosforilase e da sintase, explicando a natureza do processo e as enzimas
envolvidas (no considere o controle hormonal).

74

SNTESE E DEGRADAO DE CIDOS GRAXOS


1) A via catablica de degradao de cidos graxos para produo de ATP ocorre na matriz
mitocondrial e se chama beta-oxidao. Esta via leva clivagem sequencial da cadeia do cido
graxo em pares de C, liberando a cada ciclo: 1 acetil-CoA, 1 NADH e 1 FADH2 (ver reaes
abaixo), que alimentaro, respectivamente, o ciclo de Krebs e a cadeia respiratria. Mas, a betaoxidao exige previamente uma ativao inicial que consome 1 ATP e libera o cido graxo na
forma de acil-CoA. Esta etapa preliminar de ativao se d associada membrana externa da
mitocndria e, a transferncia da acil-Coa para dentro da mitocndria, mediada pela carnitina.
Reaes de um ciclo de beta-oxidao:
O
R CH2 CH2 CH2 C S-CoA

(acil-CoA)

oxidao

FAD
FADH2
H

R CH2 C = C C S-CoA

(enoil-CoA)

H
hidratao

H2O

HO H

R CH2 C C C S-CoA
H

(L-hidroxiacil-CoA)

H
NAD+

oxidao

NADH
O

R CH2 C CH2 C S-CoA


tilise

(ceto acil-CoA)
CoA

O
R CH2 C S-CoA +

O
H3C C S-CoA

(acetil-CoA)

2. A oxidao completa de uma molcula de palmitato (16 C) a CO2 e H2O, atravs da betaoxidao, ciclo de Krebs e cadeia respiratria, rende 129 ATP. importante destacar que este
rendimento, medido em ATP/mol-oxidado, muito superior ao da oxidao completa de

75

aucares e protenas, pois a oxidao de um cido graxo leva liberao de 37,6 kJ/g de
energia livre, enquanto oxidao de aucares ou protenas libera apenas 16,7 kJ/g.
3. Os cidos graxos so sintetizados no citosol por via anablica prpria que adiciona
seqencialmente unidades de 2 C cadeia em crescimento. Esta via alimentada por acetilCoA, mas s a primeira unidade de 2 C entra como acetil-CoA, as subseqentes so na forma
de malonil-CoA. Portanto, o acetil-CoA precisa ser previamente ativado a malonil-CoA, por
carboxilao e consumo de 1 ATP, para permitir a reao de condensao, levando ao
crescimento da cadeia do cido graxo de uma unidade de 2 C, por ciclo de sntese. A ativao
de acetil-CoA catalisada pela acetil-CoA Carboxilase, uma enzima sujeita a controle complexo,
envolvendo regulao alostrica e ativao / desativao por modificao covalente (fosforilao
/ desfosforilao).
4. A sntese de palmitato (16 C) altamente endergnica, obedecendo a sequinte estequiometria:
Acetil-CoA + 7 malonil-CoA + 14 NADPH + 7H+ palmitato + 7 CO2 + 14 NADP+ + 8 CoA +
6H2O
A elongao da cadeia alm de 16 C e a insero de duplas ligaes feita por outros sistemas
enzimticos especializados, que se localizam na membrana do retculo endoplasmtico. Mas,
mamferos no possuem enzimas para introduzir duplas ligaes em cadeias de cidos graxos
acima do C9. Por isso, linoleato (18:2) e linolenato (18:3), so cidos graxos essenciais que
precisam ser adquiridos pela dieta.
5. importante destacar que animais degradam eficientemente glicose at acetil-CoA pela gliclise
e assim podem converter C de acar em cadeias de lipdeo de reserva. Mas, estes organismos
no podem fazer o caminho de volta de cadeias de cido graxo para glicose, pois no possuem
reaes que convertam acetil-CoA em piruvato ou oxalacetato.

Exerccios 19
1) Ponto de fuso dos cidos graxos. Os pontos de fuso de uma srie de cidos graxos de 18
tomos de carbono so: cido esterico (69,9oC), cido olico (13,4oC), cido linolico (-5oC) e
cido linolnico (-11oC). Que aspecto estrutural destes cidos graxos de 18 carbonos pode ser
correlacionado com o ponto de fuso? Fornea uma explicao molecular para esta tendncia do
ponto de fuso.
2) Mostre como os cidos graxos so ativados antes de serem degradados. Em que compartimento
celular isso ocorre?

76

3) Diga onde e sob que forma so estocados os cidos graxos? Como os cidos graxos da reserva
metablica so mobilizados para serem oxidados na matriz mitocondrial?
4) Explique os passos da -oxidao dos cidos graxos, partindo de uma molcula de palmitato.
Calcule o rendimento energtico da oxidao completa de palmitato a CO2 e H2O.
5) Na -oxidao, a cadeia de cidos graxos degradada aos pares de carbono. Na sntese de
cidos graxos, a cadeia cresce tambm aos pares de carbono. No entanto, o precursor na
elongao da cadeia, durante a sntese, malonil-CoA e no acetil-CoA. Explique porqu.
6) Equacione as etapas que compem o conjunto de reaes que permitem adicionar acetil-CoA
cadeia de cido graxo crescente durante a sntese de palmitato. Mostre que etapa torna o processo
favorecido termodinamicamente.
7) Compare a -oxidao e a biossntese de palmitato, mostrando diferenas e semelhanas em:
i. carregadores de grupos acila;
ii. reaes de xido-reduo;
iii. coenzimas de xido-reduo;
iv. gasto ou produo de energia em termos de equivalentes de ATP e de coenzimas redutoras.
8) Mostre como se d a oxidao do cido olico.

77

METABOLISMO DE AMINOCIDOS
1. A principal fonte de nitrognio para os seres vivos o N2 atmosfrico, que para ser
assimilado pelas clulas, precisa primeiramente, ser convertido em amnio. A converso de
N2 em NH3 chamada de fixao de nitrognio. Somente bactrias podem fixar nitrognio.
2. Cerca de 70% do nitrognio disponvel para os seres vivos so fixados por bactrias. Outros
25% so produzidos industrialmente (fertilizantes) e 10-15% so fixados pela natureza por
processos no biolgicos, tais como descargas eltricas (raios) e radiao ultravioleta.
3. A amnia produzida pela fixao do nitrognio inicialmente utilizado para produzir
glutamato e glutamina. O grupo amino dos outros aminocidos obtidos a partir desses
precursores.
4. Humanos so capazes de sintetizar apenas 11 dos 20 aminocidos necessrios para a vida
(denominados de aminocidos no essenciais). Portanto, 9 aminocidos devem ser obtidos a
partir da dieta e so denominados de aminocidos essenciais. Bactrias e microorganismos,
so capazes de sintetizar todos os 20 aminocidos. Cistena e tirosina, apesar de serem
considerados no essenciais, so sintetizados em humanos a partir de precursores obtidos
de aminocidos essenciais (metionina e fenilalanina, respectivamente).
5. A sntese dos aminocidos no essenciais pode ser dividida em 5 grupos, de acordo com o
precursor de seu esqueleto de carbono: (1) alfa-cetoglutarato, (2) oxaloacetato, (3) piruvato,
(4) 3-fosfoglicerato e (5) fenilalanina.
6. Durante o ciclo de vida normal de uma clula, protenas so constantemente sintetizadas e
degradadas, e alguns dos aminocidos liberados so degradados para a gerao de energia
(catabolizados). H dois processos principais para a degradao de protenas: pela ao de
proteases lisosomais ou (2) pela marcao com ubiquitina e posterior degradao pelo
sistema do proteosoma.
7. Adultos numa dieta normal obtm 10-15% de suas necessidades energticas atravs da
degradao de aminocidos. Quando a dieta rica em protenas e a ingesto de
aminocidos excede a necessidade do organismo, esses aminocidos so catabolizados
para gerar energia. Protenas (aminocidos) no podem ser estocados como fonte de
reserva de energia ou aminocidos.
8. No jejum ou diabetes no controlada, condies onde os nveis de glicose esto abaixo do
normal, ou indisponveis com fonte de energia, protenas celulares so degradadas e seus
aminocidos utilizados como fonte de energia.
9. O processo de catabolismo do aminocido se inicia com a remoo do grupo amino, seguido
pela oxidao do esqueleto de carbono do alfa-cetocido em H2O e CO2.
10. O remoo do grupo amino da maioria dos aminocidos se d pela transferencia para o alfacetoglutarato, formando glutamato mais o alfa-ceto-aminocido correspondente. Essa

78

reao catalisada por uma aminotransferase (tambm chamadas de transaminases). As


aminotransferases so encontradas no citossol e na mitocndria, e contm o piridoxal-fosfato
como cofator. O piridoxal-fosfato derivado da vitamina B6 (piridoxina).
11. O grupo amino do glutamato pode ter dois destinos: (1) ser transferido do glutamato para o
oxaloacetato, formando aspartato e alfa-cetoglutarato, ou (2) ser removido pela glutamato
desidrogenase, gerando amnia e alfa-cetoglutarato. A glutamato desidrogenase um
enzima rara, pois utilizar tanto o NAD+ quanto o NADP+ como coenzima.
12. Nove aminocidos no so substratos de aminotransferases. O grupo amino desses
aminocidos Asp, Gly, Glu, His, Lys, Met, Pro, Ser e Thr removido por reaes
particulares para cada aminocido. Porm, em todos os casos, os produtos finais tambm
so o aspartato ou a amnia, ambos precursores do ciclo da uria.
13. O esqueleto de carbono dos aminocidos degrado a piruvato, acetil-CoA ou intermedirios
do ciclo de Krebs, servindo portanto, como fonte de energia. Dependendo do aminocido e
do estado metablico do organismo, os aminocidos podero ser (1) oxidados no ciclo de
Krebs, fornecendo energia, (2) utilizados para a sntese de glicose atravs neoglicognese,
ou (3) convertidos em triacilgliceris para armazenamento.
14. O processo de degradao do esqueleto de carbono pode ser dividido em seis grupos, de
acordo com o produto final: 1) Piruvato, 2) Oxaloacetato, 3) Fumarato 4) Succinil-CoA, 5)
alfa-cetoglutarato e 6) Acetil-CoA.
15. A maioria dos aminocidos produzem piruvato ou intermedirios do ciclo de Krebs e so
chamados de glicognicos. Leucina e lisina produzem, exclusivamente, corpos cetnicos a
partir do Acetil-CoA, e so denominados cetognicos. Alguns aminocidos so tanto
glicognicos como cetognicos, pois seus esqueletos carbnicos servem como fonte para a
produo de Acetil-CoA ou Aceto-Acetil-CoA, assim como para intermedirios do ciclo de
Krebs.

Exerccios 20

1. Definir aminocido essencial e citar os aminocidos essenciais para o homem.


2. Qual a primeira etapa do catabolismo de aminocidos e quais so as enzimas que
participam.
3. Esquematizar as reaes catalisadas pelas seguintes enzimas: aspartato aminotransferase
(glutmico-oxaloactico transaminase - GOT) e alanina aminotransferase (glutmico-pirvico
transaminase - GTP). Citar a coenzima que participa das reaes e a vitamina presente na
sua estrutura.
4. Verificar o destino dos esqueletos de carbono dos aminocidos em seu catabolismo e indicar
aqueles que podem originar glicose.

79

5. Quais as conseqncias do defeito gentico que causa a inativao da fenilalanina


hidroxilase?

80

CICLO DA URIA
1. A amnia txica para o organismo e para ser removida da clula e secretada
eficientemente, precisa primeiro ser metabolizada sob a forma de uria. Aves e rpteis
excretam cido rico, e peixes, amnia. Em mamferos terrestres, a uria sintetizada no
fgado a partir de amnia, aspartato e CO2 (ciclo da uria): um tomo de nitrognio da uria
provm da amnia enquanto o outro provem do aspartato. Um indivduo adulto produz e
secreta cerca de 30g de uria por dia.
2. O ciclo da uria acontece na matriz mitocondrial e no citossol. O glutamato e glutamina so
transportados para a mitocndria, onde so desaminados, gerando amnia e alfacetoglutarato.
3. Na mitocndria, o amnio se liga ao bicarbonato formando carbamoil fosfato numa reao
catalisada pela enzima carbamoil fosfato sintetase. O carbamoil fosfato condensa-se com a
ornitina, formando citrulina, que ento transportada para o citossol.
4. No citossol, a citrulina se une ao aspartato gerando fumarato e arginina. A arginina ento
hidrolisada produzindo uria e regenerando a ornitina, que retorna mitocndria para
reiniciar o clico da uria.
5. O grupo amino removido dos aminocidos na forma de glutamato ou amnia precisa ser,
primeiro, transportado para o fgado antes de ser metabolizado no ciclo da uria.
6. A amnia e o glutamato so convertidas em glutamina pela ao da glutamina sintetase. A
glutamina ento transportada pela corrente sanguneo para o fgado, aonde, na
mitocndria heptica, novamente convertida em

amnia e glutamato pela enzima

glutaminase.
7. No msculo, o grupo amino transportado para o fgado atravs do ciclo da glicose-alanina,
onde a alanina produzida no msculo a partir do piruvato, serve de carregadora para o grupo
amino. No fgado, o grupo amino da alanina transferido para o alfa-cetoglutarato, formando
glutamato.
8. O ciclo da uria e o ciclo do cido ctrico (Krebs) so interconectados atravs do fumarato,
formando

chamada

bicicleta

de

Krebs.

Porm,

ambos

os

ciclos

operam

independentemente um do outro, e a comunicao entre eles dependem do transporte de


compostos intermedirios atravs da membrana mitocondrial.
9. A regulao do ciclo da uria se d por uma ao estimulatria alostrica do N-acetilglutamato na enzima responsvel pela primeira etapa do ciclo: a carbamoil fosfato sintetase.
O N-acetil-glutamato formado pela unio do Acetil-CoA com o glutamato, atravs da
enzima N-acetil glutamato sintase. Essa enzima, por sua vez, estimulada alostericamente
pela arginina, um dos produtos do ciclo da uria.
10. Pessoas com deficincia em alguma das enzima do ciclo da uria no conseguem
metabolizar eficientemente a amnia produzida no fgado, resultando em hiperamonemia,

81

com srias conseqncias para o organismo. Como humanos so incapazes de sintetizar 9


dos 20 aminocidos, uma dieta livre de protenas no um tratamento possvel. Vrias
dietas esto disponveis para indivduos com deficincia no ciclo da uria, dependendo da
enzima afetada.

Exerccios 21

1. Como se explica bioquimicamente a toxidez da amnia?


2. Qual a origem dos tomos de nitrognio da uria?
3. Qual a equao final da sntese de uria? Qual , no cmputo geral, o gasto energtico
necessrio para o organismo (ligaes de fosfato) sintetizar a uria? Qual o aminocido
proteico sintetizado?
4. Explique como a velocidade do ciclo da uria regulada.
5. Explique porque a deficincia parcial do ciclo da uria atenuada em condies de uma
dieta com baixa quantidade de protenas.
6. Explique como uma baixa velocidade do ciclo de uria pode interferir com o ciclo de Krebs.
7. Animais, de forma geral, no possuem reservas na forma de protena ou qualquer outra
macromolcula nitrogenada. Quais as conseqncias desse fato para o balano de
nitrognio nesses organismos em condies de alimentao abundante e de jejum
acentuado?
8. Um adulto normal, com uma dieta desprovida de protenas, elimina uria. Por que?
9. Definir balano de nitrognio e citar as condies que levam a um balano positivo ou
negativo de nitrognio.
10. Analisar o destino da cadeia carbnica dos aminocidos e o balano de nitrognio que
ocorrem com as seguintes dietas:
11. normal em carboidratos, lipdios e protenas.
12. rica em protenas e normal nos demais componentes.
13. pobre em carboidratos e normal nos demais componentes.
14. pobre em protenas e normal nos demais componentes.
15. rica em protenas deficientes em um aminocido essencial e normal nos demais
componentes.

82

METABOLISMO DE PURINAS E PIRIMIDINAS


1. Os nucleotdeos de purina so degradados atravs de uma via na qual o grupo fosfato
liberado pela ao da 5-nucleotidase. O adenilato libera adenosina, que ento desaminada
a inosina pela adenosina desaminase. A inosina sofre hidrlise para liberar D-glicose e a
base purnica hipoxantina. Esta ltima oxidada sucessivamente a xantina e cido rico pela
xantina oxidase.
2. O catabolismo do GMP (guanilato) tambm libera cido rico como produto final. O GMP
inicialmente hidrolisado para liberar o nucleosdeo guanosina, o qual clivado para liberar
guanina. A guanina sofre remoo hidroltica do seu grupo amino, liberando xantina, que
convertida em cido rico pela xantina oxidase.
3. Embora o cido rico seja o produto final do catabolismo das purinas nas aves, primatas,
rpteis e insetos, em muitos outros organismos, ele ainda mais degradado para o produto
de excreo alantona, sob a ao da urato oxidase. Os peixes cartilaginosos e os anfbios
degradam a alantona uria antes da excreo. Finalmente, invertebrados marinhos
decompem a uria a seu produto de excreo nitrogenado, NH4+.
4. A gota uma doena provocada por nveis elevados de cido rico no sangue e nos tecidos.
E, devido a um depsito anormal de cristais de urato de sdio, as articulaes se inflamam,
tornando-se dolorosas e artrticas. Os rins so afetados porque o cido rico tambm se
deposita nos tbulos renais. A gota pode ser tratada pela administrao do inibidor da
xantina oxidase alopurinol.
5. As clulas animais degradam os nucleotdeos de pirimidina a seus componentes base. A
timina e uracila resultantes so ento clivadas no fgado atravs de reaes de reduo. Os
produtos do catabolismo das pirimidinas, -alanina e -aminoisobutirato so aminocidos e,
portanto, metabolizados como tais. So convertidos, atravs de reaes de transaminao e
ativao, a malonil-CoA e metilmalonil-CoA. Este ltimo, por sua vez, convertido em
succinil-CoA para posterior utilizao.

Exerccios 22
1. Que vantagem alguns organismos levam por excretar preferencialmente cido rico e no
uria como o produto final do catabolismo das purinas?
2. Por que o cido rico txico para humanos?
3. Descreva, resumidamente, os cuidados alimentares que um portador de gota deve tomar,
justificando cada caso. Explique, tambm, por que o alcoolismo crnico pode agravar o
quadro patolgico da hiperuricemia.
4. Um determinado indivduo, aps radioterapia e tratamento com mercaptopurina, um
antineoplsico, apresentou hiperuricemia e outros sintomas caractersticos da gota. Quando

83

o alopurinol foi administrado, os sintomas no s desapareceram como houve um aumento


na eficcia do tratamento com mercaptopurina. Explique bioquimicamente.

84

FOTOSSNTESE
1. A fotossntese o processo pelo qual a energia luminosa transformada em energia qumica
e poder redutor, armazenados nas molculas de ATP e NADH +H+. Num segundo passo,
chamado de fase escura (mas na realidade, fase independente da luz), a energia
armazenada utilizada para sintetizar glicose a partir de CO2 e H2O. A fotossntese ocorre
nos cloroplastos, uma organela que, como a mitocndria, possui uma membrana externa
altamente permevel e uma membrana interna praticamente impermevel, separadas por um
espao intermembranar.
2. As reaes dependentes da luz ocorrem na membrana tilacide e envolvem processos
semelhantes ao transporte de eltrons e fosforilao oxidativa da mitocndria. As reaes
independentes de luz ocorrem no estroma.
3. Os primeiros estudos de fotossntese realizados levaram concluso de que o CO2 era a
fonte do O2 produzido pela fotossntese. Em 1931, entretanto, demonstrou-se que bactrias
fotossintetizantes anaerbicas, sintetizam glicose a partir de CO2, sem gerar O2:
CO2 + 2 H2S luz (CH2O) + 2 S + H2O
A reao geral da fotossntese pode ser demonstrada como segue:
CO2 + 2 H2A luz (CH2O) + 2 A + H2O
Em cianobactrias, H2A H2S, e em plantas, H2O. Isso sugere que a fotossntese seja
um processo de duas fases, nos quais a energia solar utilizada para oxidar H2A (fase
clara):
2 H2A luz 2 A + 4[H]
e o agente redutor resultante [H] subsequentemente reduz CO2 (fase escura):
4[H] + CO2

luz

(CH2O) + H2O

4. O principal fotorreceptor na fotossntese a clorofila. A luz absorvida pelas clorofilas antena


e pigmentos acessrios transferida para centros de reao fotossintticos, onde ocorrem
as principais reaes da fotossntese.
5. Plantas e cianobactrias utilizam o poder redutor gerado pela oxidao de H2O dirigida pela
luz para produzir NADPH.
6. A produo de O2 na fotossntese requer 2 fotossistemas: Fotossistema I (P700) gera um
forte agente redutor, capaz de reduzir NADP+, e concomitantemente, um oxidante fraco;
Fotossistema II (P680) gera um forte agente oxidante, capaz de oxidar H2O, e
concomitantemente, um redutor fraco. O redutor fraco reduz o oxidante fraco. Assim,
fotossistemas I e II precisam funcionar em srie para acoplar a oxidao da H2O com a

85

reduo de NADP+ (transferncia de eltrons de H2O para NADP+, formando O2 e NADPH +


H+).
7. Quando iluminado, o FS II passa para uma forma excitada e perde eltrons, os quais so
transportados por reaes de xido-reduo para o fotossistema I. O PS I iluminado fornece
eltrons para a reduo de NADP+. Como resultado temos a oxidao do FS II e a reduo
do FS I. A reposio de eltrons em PS II feita por eltrons provenientes da oxidao de
gua e, em PSI, por eltrons emitidos por PSII.
8. Os componentes envolvidos no transporte de eltrons de H2O para NADP+ com produo de
NAPDPH + H+ esto organizados em trs partculas, que esto ligadas a membrana
tilacide: (1) fotossistema II; (2) complexo do citocromo b6f; (3) fotossistema I
(fotofosforilao no-ciclica).
9. Os cloroplastos geram ATP de maneira muito semelhante da mitocndria, ou seja, atravs
do acoplamento da dissipao de um gradiente de prtons sntese de ATP.
10. Na fotofosforilao cclica, os eltrons emitidos por P700 (PSI) so transferidos ao complexo
citocromo b6f, retornando finalmente a P700. No h sntese de NADPH + H+, nem liberao
de oxignio.
11. Na fase clara da fotossntese, ATP e NADPH + H+ so sintetizados, e esses so utilizados na
fase escura para a sntese de carboidratos. A via pela qual as plantas incorporam CO2 em
carboidratos denominada de Ciclo de Calvin.
12. O ciclo de Calvin engloba duas fases: (1) a fase de produo, na qual 3 molculas de
ribulose-5-fosfato reagem com 3 molculas de CO2, gerando 6 molculas de gliceraldedo-3fosfato, com o gasto de 9 ATPs e 6 NADPH + H+; (2) a fase de recuperao, na qual os
tomos de carbono de 5 gliceraldedo-3-fosfato entram em uma srie de reaes para dar
origem a 3 ribulose-5-fosfato, com as quais o ciclo recomea.

Exerccios 23

1. Porque as folhas das plantas so verdes? Explique em termos de composio e espectro de


absoro dos pigmentos foliares.
2. O que fotofosforilao cclica e no que difere da fotofosforilao no cclica? Quando o
[NADPH + H+ / NADP+] alto, a produo de O2 durante fotofosforilao suprimida.
Explique o fenmeno considerando os papis dos dois fotossistemas.
3. Compare a fase clara da fotossntese com o processo de respirao na mitocndria,
indicando diferenas e semelhanas.
4. Sabe-se que a produo de O2 na fotossntese requer o funcionamento de dois
fotossistemas. Explique resumidamente como ocorre a interao entre os dois fotossistemas,
incluindo doador e receptor de eltrons no processo.

86

5. Sabe-se que a produo de O2 na fotossntese requer o funcionamento de dois


fotossistemas. Explique resumidamente como ocorre a interao entre os dois fotossistemas,
incluindo doador e receptor de eltrons no processo.
6. Explique a regulao do ciclo de Calvin, mencionado as enzimas-chave e como as reaes
de cada fase controlam o processo de fixao do CO2.

87

INTEGRAO DO METABOLISMO
1. O fluxo metablico deve ser controlado para haver equilbrio entre o fornecimento e a demanda
de nutrientes e para manter homeostasia. A integrao do metabolismo em termos do

organismo completo requer uma reavaliao integrativa dos conceitos estudados em


cada via metablica individual. Esta avaliao inclui um entendimento da especializao
de cada rgo ou tecido e dos sinais hormonais que indicam e coordenam estas aes.
2. O fluxo metablico atravs de uma etapa determinante da velocidade de uma via pode ser
alterado por controle alostrico, modificao covalente ou controle da enzima que catalisa a
reao. freqente que um produto final atue como efetuador alostrico negativo de uma
enzima que catalisa uma das primeiras reaes da via (retroinibio); esse mesmo produto final
pode atuar como efetuador alostrico positivo em outra via que o utilize como substrato inicial.
3. As etapas determinantes da velocidade de uma via so endergnicas.
4. A funo central do fgado no metabolismo processar e distribuir nutrientes. Ele transforma os
nutrientes obtidos da dieta em combustveis e precursores requeridos por outros tecidos e os
exporta para o sangue. O metabolismo de glicose controlado por insulina e glucagon. A
sntese do glicognio catalisada pela glicognio-sintetase e a degradao catalisada pela
glicognio-fosforilase. A glicognio-sintetase tem duas formas: fosforilada (inativa) e nofosforilada (ativa). A glicognio-fosforilase possui duas formas: fosforilada (ativa) e nofosforilada (inativa) (ver captulos glicognio e hormnios e diabetes). O glucagon promove a
ativao da fosforilase quinase, causando o estmulo da degradao do glicognio e inibindo a
sntese dele da mesma forma que ocorre no msculo.

A insulina promove a ativao da

fosfatase 1, levando a sntese de glicognio e inibindo sua degradao da mesma forma que
ocorre no msculo. A glicognio-fosforilase inativa pode ser estimulada por AMP e, na forma
ativa, pode ser inibida pela glicose.
5. No fgado, a regulao da gliclise e da gliconeognese feita de forma recproca (quando uma
via est ativa, a outra desacelerada). Os principais pontos de controle da gliclise so:
converso de glicose a glicose-6-fosfato, de frutose 6-fosfato a frutose 1,6-bifosfato e de
fosfoenolpiruvato a piruvato (reaes irreversveis).
6. O controle da gliclise e da gliconeognese no fgado mediado pela frutose-2,6-bifosfato, que
ativa a fosfofrutoquinase-1, estimulando a gliclise no fgado, e inibe a frutose-1,6-bifosfatase,
inibindo a gliconeognese. A concentrao celular de frutose-2,6-bifosfato regulada pelas
velocidades relativas de sua sntese, catalisada pela fosfofrutoquinase-2, e sua destruio,
catalisada pela frutose-2,6-bifosfatase. Fosfofrutoquinase-2 e frutose-2,6-bifosfatase so
atividades diferentes de uma nica protena; essa protena, quando fosforilada, tem sua
atividade frutose-2,6-bifosfatase aumentada e sua atividade fosfofrutoquinase-2 diminuda. O

88

glucagon causa a fosforilao da protena e, dessa forma, ele inibe a gliclise e estimula a
gliconeognese.
7. Mais um stio de controle da gliclise e da gliconeognese incide sobre a piruvato quinase, que
converte fosfoenolpiruvato em piruvato; Piruvato quinase est ativa quando desfosforilada e
inativa quando fosforilada. A via de sinalizao do glucagon leva a fosforilao da piruvato
quinase e conseqente inativao, enquanto a insulina promove a desfosforilao da piruvato
desidrogenase e, portanto, sua ativao.
8. No fgado, a piruvato quinase inibida por alanina; essa inibio fundamental para levar o
fgado a converter o aminocido em glicose (por gliconeognese) e no em piruvato; a alanina,
oriunda do msculo, convertida em glicose no fgado e volta para o msculo novamente em
forma de glicose;
9. No msculo, a adrenalina ou o aumento intracelular de clcio promove a ativao da fosforilase
quinase, que fosforila a glicognio sintetase, inativando-a, e fosforila a glicognio fosforilase,
ativando-a, o que estimula a degradao do glicognio e inibe a sua sntese.
10. Na maioria dos tecidos, a fosforilao da glicose catalisada pela hexoquinase, que inibida
pelo seu produto, a glicose-6-fosfato; esse mecanismo de inibio ajusta a captao da glicose
sua utilizao.
11. No fgado, a fosforilao da glicose catalisada pela glicoquinase, que tem Km maior que o da
hexoquinase e no inibida pelo seu produto; o Km alto da glicoquinase impede a captao de
glicose pelo fgado quando a glicose est em baixas concentraes no sangue.
12. Quando a concentrao de glicose no sangue est abaixo da necessria, a glicose-6-fosfato
presente no fgado desfosforilada para a produo de glicose livre, que exportada para o
sangue;
13. No h regulaes por modificao covalente conhecidas para o ciclo de Krebs. NADH age
como inibidor alostrico das enzimas do ciclo de Krebs;

A isocitrato desidrogenase, que converte isocitrato em -cetoglutarato, estimulada por ADP e


inibida por NADH;

O complexo -cetoglutarato desidrogenase, que converte -cetoglutarato em succinil-CoA,


inibido por succinil-CoA, NADH e ATP;

A citrato sintase inibida competitivamente por succinil-CoA;

A succinato desidrogenase inibida competitivamente por oxaloacetato;

14. Enzimas da sntese e degradao de triacilgliceris so fosforilados e defosforilados por


transduo de sinal induzida por glucagon, adrenalina e insulina. A lipase, enzima dos adipcitos
que hidrolisa triacilgliceris, ativada por fosforilao e inativada por desfosforilao. Glucagon
e adrenalina promovem a fosforilao da lipase, causando a degradao de triacilgliceris. A
insulina promove a desfosforilao da lipase, inibindo a degradao de triacilgliceris;
15. A sntese de cidos graxos e, conseqentemente, a de triacilgliceris, pelo fgado de adipcitos
tem como principal ponto de regulao a reao catalisada pela acetil-CoA carboxilase
(converso de acetil-CoA a malonil-CoA). A acetil-CoA carboxilase constitudas por protmeros

89

sem ao cataltica que, ao se associarem, passam a ter funo cataltica. A presena de


palmitoil-CoA estimula a desagregao desse protmeros, como tambm a citrato liase, a
tricarboxilato translocase (enzima que transfere citrato para o citosol) e a piruvato
desidrogenase;
16. Acetil-CoA carboxilase tambm est sujeita a regulao por modificao covalente: glucagon e
adrenalina promovem a fosforilao e conseqente inativao da enzima, enquanto a insulina
promove a desfosforilao e conseqente ativao da enzima. A insulina estimula a sntese de
lipdios de outras formas: estmulo da piruvato desidrogenase, da glicerol 3-fosfato acil
transferase e da expresso das enzimas acetil-CoA carboxilase, sintase de cidos graxos e
lipase lipoprotica. Malonil-CoA inibe a carnitina acil transferase I, evitando a entrada dos
cidos graxos recm sintetizados na mitocndria e sua conseqente oxidao.
17. Os ajustes metablicos desencadeados por insulina, glucagon e adrenalina ocorrem em escala
curta de tempo; a leptina uma protena que regula o metabolismo em uma escala de tempo
mais longa;
18. A leptina produzida nos adipcitos e atua nos receptores do hipotlamo, causando diminuio
do apetite. A produo e liberao da leptina aumentam com o nmero e o tamanho dos
adipcitos. A leptina promove, nas mitocndrias dos adipcitos, o desacoplamento entre
transporte de eltrons e fosforilao oxidativa, causando termognese. A liberao de leptina
pelo tecido adiposo reduzida durante os perodos de jejum.
Exerccios 24

1. Os nutrientes absorvidos no trato intestinal passam diretamente ao fgado (com exceo de uma
grande parte de triacilgliceris). Por este grande centro de distribuio, acares, aminocidos e
alguns lipdeos so processados e distribudos aos outros rgos e tecidos. Propor um mapa
metablico unificado, integrando as principais vias destes nutrientes neste rgo.
2. Ao contrrio do msculo esqueltico, o msculo cardaco deve trabalhar constante e
ritmicamente.

que

torna

possvel,

em

termos

metabolicos

caractersticas

funcionais/estruturais da clula cardaca, esta adaptao para o trabalho constante?


3. O glicerol-3-fosfato um intermedirio chave na biossntese dos triacilgliceris. As clulas
adiposas, que so especializadas na sntese e degradao dos triacilgliceris, no podem usar o
glicerol diretamente devido falta da gliceroquinase, que catalisa a reao:
Glicerol + ATP Glicerol-3-fosfato + ADP
Como o tecido adiposo obtm o glicerol-3-fosfato necessrio para a sntese do triacilglicerol?
Explicar.
4. Pacientes com deficincia de tiamina exibem um conjunto de sinais neurolgicos caractersticos:
perda dos reflexos, estado de ansiedade, e confuso mental. Sugira uma razo por que a
deficincia de tiamina manifestada na funo cerebral.

90

5. Pessoas que consomem muita bebida alcolica desenvolvem nveis sangneos no normais e
baixos de glicose (hipoglicemia), maiores do que os normais de cido lctico (acidose lctica suave)
e ainda apresentam cetose. Explique.

91

FLUXO DE MATERIAIS EM HUMANOS E TRANSPORTE EM MEMBRANAS


1. A digesto tem duas finalidades principais: (1) transformar os principais grupos de nutrientes
(carboidratos, protenas e lipdeos) em seus componentes fundamentais capazes de
atravessar a membrana celular e serem assim absorvidos; (2) evitar que estes nutrientes
penetrem na clula ntegros e se comportem como substncias estranhas ao organismo.
Quando um polissacardeo ou protena so ingeridos, estes no podem atravessar a mucosa
intestinal por serem molculas grandes s quais a membrana impermevel. Normalmente,
na luz do aparelho digestrio, estes polmeros sero digeridos por enzimas adequadas que
os transformaro em glicose e aminocidos.

2. A digesto nos humanos consiste na secreo de enzimas responsveis pela hidrlise de


substratos cujos componentes sero posteriormente incorporados ao organismo e levados
s suas vias metablicas correspondentes.
3. As membranas so barreiras hidrofbicas que oferecem grande resistncia passagem de
solutos hidroflicos, cuja permeao exige protenas transportadoras especficas, conforme
esquematizado na figura abaixo. Desta maneira a membrana, atravs de transportadores
especficos, regula o transporte de metablitos entre compartimentos celulares.

5. Um exemplo clssico de transporte a tomada de glicose pela hemcia mediada por um


transportador especfico, cuja velocidade depende da concentrao externa de glicose e
obedece a uma curva hiperblica de saturao j bem conhecida da cintica enzimtica,
sendo Kt anlogo a Km.

92

Esta forma de transporte conhecida como transporte passivamente mediado ou difuso


facilitada. Trata-se de um processo exergnico, pelo qual o soluto, no caso a glicose,
atravessa espontaneamente a membrana indo do compartimento de maior para o de menor
concentrao.
6. Existem 5 transportadores conhecidos que medeiam a difuso facilitada de glicose em
humanos: GLUT1 a 5, cujos Kts so diferentes para atender as necessidades funcionais dos
tecidos nos quais so expressos. GLUT1 o transportador em hemcias, j GLUT2
expresso no fgado e clulas beta do pncreas, enquanto GLUT4 aparece no msculo
esqueltico, tecido adiposo etc.
7. Mas, no epitlio do intestino a glicose obtida da dieta transportada para dentro da clula
contra o gradiente de concentrao, portanto atravs de um processo endergnico que exige
consumo de energia metablica para ocorrer e referido como transporte ativo. Neste caso o
transportador chamado simporte, pelo qual a glicose transportada junto com Na+ e
termodinamicamente possvel porque existe um gradiente eletroqumico de Na+ de fora para
dentro da clula. H mltiplas formas de transporte ativo, das quais este exemplo da glicose
apenas uma delas. Grande parte da energia metablica consumida pelas clulas se deve
manuteno da enorme diversidade de transportadores que promovem a transferncia de
metabolitos e ons contra gradientes de concentrao.

Exerccios 25
1. Descreva, resumidamente, como ocorre a digesto de carboidratos, protenas e lipdeos,
apontando os locais onde acontece e as enzimas envolvidas.
2. Como se d a absoro de carboidratos, aminocidos e lipdeos.
3. Para saber se uma bactria tomava leucina e etilenoglicol por transporte mediado ou no
mediado, foram feitas medidas de velocidade inicial de tomada em funo da concentrao
de ambas substncias, resultando na tabela fornecida abaixo. O que voc conclui do exame
dessa tabela? Explique e calcule Kt e Vmax se encontrar evidncias de transporte mediado.
Componente
Leucina

Concentrao [M]

Velocidade Inicial (unidades arbitrrias)

-6

110

-6

220

-6

480

-5

830

-5

1700

-4

2600

-4

3100

-3

3200

1 x 10
2 x 10
5 x 10
1 x 10
3 x 10
1 x 10
5 x 10
1 x 10

93

Etileno glicol

1 x 10-3

-3

5 x 10

0,01

10

0,05

50

0,1

100

0,5

500

1,0

1000

4. Clulas epiteliais de intestino de camundongo isoladas em cultura transportam L-leucina e Dleucina mostrando Kt (mM) e Vmax, respectivamente iguais a: 0,24 e 420 para L-leucina e 4,7
e 310 para D-leucina, ambos em presena de Na+ no meio de cultura. Mas na ausncia de
Na+, L-leucina mostra 0,24 e 23 enquanto D-leucina mostra 4,7 e 5 para Kt (mM) e Vmax,
respectivamente. Classifique esse transportador de leucina quanto ao tipo e mecanismos de
ao. Que efeitos voc esperaria se nesse meio de cultura fosse colocada valinomicina
(ionforo de Na+)? E se fosse dissolvida ouabana (inibidor da ATPase Na+/K+) no meio de
cultura? Explique.
5. O pH e a absoro de drogas. A droga aspirina, intensamente receitada, um cido fraco
com um pKa de 3,5. A aspirina absorvida para o sangue atravs das clulas de
revestimento do estmago e do intestino delgado. Para uma substncia ser absorvida ela
deve atravessar facilmente a membrana celular. A passagem atravs da membrana celular
determinada pela polaridade da molcula: molculas inicas (carregadas) e molculas
altamente polares passam lentamente, enquanto aquelas neutras e hidrofbicas passam
rapidamente. Como o pH do suco gstrico cerca de 1 e o pH no intestino delgado, cerca de
6, pergunta-se:
a) Escreva por frmulas estruturais a ionizao reversvel da aspirina.
b) Onde a aspirina mais absorvida para a corrente sangunea, no estmago ou no intestino
delgado? Justifique claramente a sua escolha.

94

CONTROLE HORMONAL E DIABETES

1. A regulao metablica feita de interferncia direta de determinadas reaes qumicas que


compem o metabolismo, aumentando ou reduzindo sua velocidade. O resultado direto deste
processo a maior oferta de substratos ou acmulo de metablitos que acabar por influenciar
outras vidas dependentes destes compostos e a forma mais eficiente de regulao desta rede
aumentar a concentrao ou alterar a eficincia da enzima.
2. Pode se controlar a sntese ou degradao enzimtica; tambm se pode modular a atividade
enzimtica atravs de mudanas conformacionais da prpria enzima provocada atravs da
ligao de compostos ou grupos na cadeia peptdica: regulao alostrica e regulao por
modificao covalente. A concentrao enzimtica tambm pode variar conforme a oferta do
substrato; alterao mediada atravs de hormnios.
3. Hormnios so sinais qumicos que permitem a comunicao entre clulas. So sintetizados em
clulas glandulares para atingir clulas alvo atravs da circulao sangunea. As clulas alvo
respondem a hormnios especficos por possurem os respectivos receptores hormonais. A
ligao do hormnio ao receptor segue uma reao de equilbrio semelhante interao
enzima-substrato: H + R [RH]: a constante de dissociao de RH (KD), correspondente
reao inversa muito baixa - (10-12 a 10-9 M) - devido alta afinidade entre hormnio e
receptor.
4. Uma parte importante dos receptores hormonais so protenas integrais de membrana, muitas
da quais tem, atualmente, sua estrutura primria conhecida e sua estrutura tridimensional
modelada, em conseqncia da clonagem e seqenciamento dos seus respectivos genes. Por
exemplo, o receptor -adrenrgico do hormnio adrenalina, encontrado em hepatcito e outros
tipos celulares, possui um peso molecular de 64 kD, compreendendo

uma nica cadeia

peptdica que, de maneira serpentiforme, atravessa a membrana 7 vezes, deixando do lado


extracelular, a extremidade N-terminal e 3 alas, e do lado intracelular, outras 3 alas mais a
extremidade C-terminal. A poro extracelular do receptor contm o stio de ligao da
adrenalina, enquanto a poro intracelular se associa a um trmero de protenas conhecidas
como protena-G, por ter um stio especfico para ligao do nucleotdeo GTP. So hoje
conhecidos mais de 1000 receptores, de mltiplos hormnios, com esta estrutura bsica
formando a superfamlia chamada dos receptores associados a protena-G. A funo deste
receptor transduzir o sinal adrenalina de fora para dentro da clula, processo que mediado
pelas protenas-G. H tambm receptores presentes no citoplasma nuclear e citoplasmtico e
neste caso, o hormnio precisa ter alta solubilidade a lipdeos, atravessando a membrana
plasmtica como os hormnios esterides para encontrar o seu receptor dentro da clula.
5. Os hormnios esto envolvidos no metabolismo em dois nveis: induo ou represso gnica de
determinadas enzimas ou atravs da modificao covalente: A fosforilao mediada pelas
protenas quinases que transferem o grupo fosfato do ATP para resduos especficos de serina,

95

treonina e tirosina, formando uma ligao ster fosfrico ou a retirada do grupo fosfato
catalisada pela ao de fosfoprotenas fosfatases atravs da hidrlise.
6. A ligao de adrenalina ao receptor -adrenrgico acoplado a protena G ativa a enzima
adenilato ciclase atravs da ativao da subunidade (por ligao de GTP), presente na face
interna da membrana citoplasmtica, qual ativa a adenilato ciclase, catalisando a formao de
cAMP a partir de ATP e desencadeando a transduo de sinal. A descoberta de cAMP, por
Sutherland e colaboradores h cerca de 40 anos, levou criao do conceito do segundo
mensageiro da ao hormonal, sendo cAMP o primeiro a ser descrito, e permitiu dar incio
progressiva compreenso dos mecanismos de ao do receptor de adrenalina. Devemos
lembrar que os primeiros mensageiros qumicos extracelulares so os hormnios. cAMP tem
efeito transiente e hidrolisada pela ao da fosfodiesterase. Na clula, o balano entre as
reaes de sntese (a) e degradao (b) regula a concentrao intracelular do cAMP.
a) ATP + H2O cAMP + 2 Pi; catalisada pela adenilato ciclase
b) cAMP + H2O AMP; catalisada pela fosfodiesterase.
7. A base da ao metablica do cAMP a ativao alostrica de uma quinase cujos substratos
so protenas, sendo conhecida como protena quinase dependente de cAMP, ou simplesmente
PKA (Protein Kinase dependent on cAMP). A PKA, uma vez ativada, catalisa a fosforilao
ativadora (modificao covalente) de uma cascata de protenas quinases que terminam na
fosforilao da fosforilase a e da sintase do glicognio, causando, respectivamente, a ativao e
a inativao dessas enzimas. O resultado final dessa seqncia de ativaes enzimticas, com
alternncia de regulao alostrica e modificao covalente, a fosforlise do glicognio
liberando glicose-1P, comandada por sinais hormonais extracelulares.
8. Os efeitos de ativao ou no da via dependem do receptor ativado, no caso dos receptores
adrenrgicos, os efeitos de 1 so mediados atravs dos ons clcio e e a ativao de 2 leva a
inibio da via de adenilato ciclase. H casos aonde a protena G do tipo Gs sendo ativadora
de adenilato ciclase e do tipo GR inibindo a adenilato ciclase. Algumas toxinas podem ativar ou
bloquear a via de transduo de sinal: toxina da clera e a toxina da coqueluche.
9. Dois hormnios so os principais responsveis pelo equilbrio da concentrao da glicose
circulante: glucagon e insulina.
10. O glucagon um hormnio que tem efeitos equivalentes aos da adrenalina no controle do
metabolismo do glicognio: possui um receptor da famlia dos receptores acoplados a protenaG e ativa a cascata que se inicia com cAMP/PKA. Este liberado em condies de hipoglicemia
ativando processos degradativos para manuteno da glicemia sangunea. A PKA (protena
quinase ativada por cAMP) fosforila a fosforilase quinase tornando-a ativa. A fosforilase quinase
fosforila agora glicognio fosforilase. A glicognio fosforilase ativada (quando fosforilada,
glicognio fosforilase b a) catalisa a hidrlise de resduos de glicose do glicognio liberando
grupos de glicose-1-fosfato. No mesmo tempo, a fosforilase quinase, ativada pela cascata do

96

receptor de glucagon-protena-G, cAMP/PKA, fosforila a glicognio sintase, a qual se torna


inativa quando fosforilada (sntase I sintase D).
11. A insulina tem efeito oposto, promove a absoro de glicose pelo fgado e msculos e usa
deposio nas reservas de glicognio. Mas importante notar que a insulina tem mecanismos
de ao totalmente diferentes da adrenalina e do glucagon. Os receptores de insulina no
pertencem famlia dos receptores acoplados a protena-G e no tm ao sobre a adenilato
ciclase. Seus receptores so do grupo de receptores cujo domnio intracelular apresenta
atividade intrnseca de protena-quinase de tirosina. A insulina estimula fosfoprotenas
fosfatases. Para reverter a ao do glucagon, a insulina promove a ativao da fosfoprotena
fosfatase que catalisa a desfosforilao da glicognio fosforilase e da glicognio sintase, levando
a inativao da primeira (fosforilase a b) e ativao da segunda (sntase D sintase I). Desta
forma o fluxo glicoseglicognio favorecido. O transporte da glicose no interior das clulas
com a atuao da insulina um processo passivo mediado por uma famlia de permeases
denominadas GLUT (glucose transporter).
12. Respostas celulares rpidas desencadeadas por hormnios s podem ser obtidas atravs da
ativao, ou da inibio, de enzimas pr-existentes. Hormnios esterides (por exemplo,
cortisol) quando secretados difundem-se pela membrana citoplasmtica e ligam-se aos seus
receptores intracelulares os quais, quando ativados, promovem no ncleo a regulao do
metabolismo pela induo da transcrio de genes que codificam enzimas especficas, levando
sntese de novo das protenas correspondentes, fenmeno conhecido como induo
enzimtica. Mas o mecanismo de induo enzimtica desencadeado por hormnios resulta
necessariamente numa resposta celular lenta, uma vez que os RNAs mensageiros (mRNAs)
precisam ser transcritos, processados, transportados para o citoplasma e finalmente traduzidos
para produzir as protenas enzimticas exigidas.
13. A adrenalina estimula uma resposta local no msculo. A liberao de adrenalina induzida por
estmulo nervoso autnomo em situaes de perigo, exerccio fsico, e hipoglicemia e induz a
degradao do glicognio com os fins de fornecer glicose-1-fosfato como fonte de energia para
atividades musculares que permitem ao animal reagir a estas situaes.
14. Regulao da gliclise e gliconeognese. A gliclise uma das vias metablicas principais para
o fornecimento de energia. No fgado, encontra-se tambm a gliconeognese, a qual , de forma
geral, uma via antagnica da gliclise. A regulao das duas vias feita de forma reciproca, isto
, quando uma delas est ativa, a outra est inibida. H trs vias sob controle metablico: as
converses reversveis de: (i) glicose para glicose-6-fosfato (hexoquinase e glicose-6-fosfatase);
(ii) frutose-6-fosfato e frutose-1,6-bisfosfato (fosfofrutoquinase e frutose-1,6-bisfosfatase; e (iii)
fosfoenolpiruvato e piruvato (piruvato quinase e fosfoenolpiruvato carboxiquinase, piruvato
carboxilase). A fosfofrutoquinase o principal ponto de regulao da gliclise. AMP e frutose2,6-bisfosfato agem como efetuadores alostricos positivos. A formao de frutose-2,6-bisfosfato
est sob controle hormonal. Em condies de hipoglicemia, o glucagon estimula a produo de

97

cAMP no fgado. Isso ativa a PKA a fosforilar e inativar a fosfofrutoquinase e ativar a frutosebisfosfatase-2, diminuindo a concentrao de frutose-2,6-bisfosfatase. Como resultado, o
equilbrio entre as reaes de fosfofrutoquinase alterado, em favor da sntese de frutose-6fosfato, aumentado o fluxo gliconeognico e a sntese de glicose-6-fosfato. Ao contrrio, em
condies de hiperglicemia, as concentraes de cAMP diminuram, e o conseqente aumento
de frutose-2,6-bisfosfato ativa a fosfofrutoquinase e promove a gliclise.
Diabetes:
1. A doena diabetes mellitus causada por uma deficincia na produo ou ao da insulina,
levando a exacerbao dos efeitos metablicos do glucagon. Como a insulina no secretada em
quantidades suficientes ou seus receptores se tornam menos sensveis ativao, a via intracelular
de transduo de sinal induzida por insulina no ativada. Consequentemente, a disfuno da
transduo de sinal induzida por insulina torna os nveis plasmticos de glicose to elevados que a
glicose tambm transborda para urina, fornecendo um teste diagnostico para doena.
2. A estimulao por glucagon qual no tem contrabalana pela insulina leva a degradao de
triacilglicerol, de cidos graxos e do glicognio como tambm a elevao de gliconeognese e
inibio da entrada de glicose nas clulas. Como efeito da inibio de entrada de glicose nas
clulas, essas praticamente morrem de fome.
3. O metabolismo acelerado de glicose e cidos graxos como tambm o aumento da
gliconogenese resulta na elevao da formao de corpos cetnicos (formado a partir de acetil
CoA) no sangue. A acumulao de corpos cetnicos acidifica o pH sanguineo. A eliminao dos
prtons em excesso leva tambm a excreo de outros ons causa hidratao grave e reduo do
volume sanguneo.
4. Existem duas formas de Diabetes mellitus:
-Diabete dependente da insulina (tipo I) causada por uma deficincia das clulas pancreticas
em produzir insulina.
-Diabete no dependente da insulina (tipo II) causada por uma deficincia dos receptores de
insulina. A escassez dos receptores de insulina pode resultar da presena continua de altas
concentraes de insulina baseada em predisposio gentica ou superalimentao. A contnua
superproduo de insulina pode inibir a sntese e/ou induzir a dessensibilizao dos seus receptores
(fazendo-os menos sensvel a estimulao por insulina).

Exerccios 26
1) Pacientes sofrendo de diabetes melittus tm, freqentemente, altas taxas de triglicerdeos no
sangue. Explique esse fenmeno.
2) Existem vrias isoformas do transportador de glicose GLUT. A isoforma GLUT1 expressa em
praticamente todos os tipos celulares; a isoforma GLUT2 expressa somente no fgado e pncreas.

98

Pensando nos papeis desses rgos no metabolismo de glicose, postule possveis diferenas entre
as caractersticas de transporte dessas duas isoformas.
3) O antibitico ionomicina um ionforo de Ca2+ que usado na clinica para induzir a produo de
citocinas, como interferon, IL-2 e IL-4. Postule um mecanismo para o controle da expresso dos
genes que codificam essas protenas.
3) Isoformas dos receptores de estrgeno ER e ER foram recentemente encontradas dentro de
mitocndrias em clulas de mamferos. Discuta o que essa observao pode sugerir?
4) Explique as diferenas entre um agonista e um antagonista de um receptor. Discuta possveis
implicaes teraputicas para agonistas e antagonistas.
5) Foi isolado um mutante da Proteina Kinase C, na qual3 aminocidos do motivo do pseudosubstrato foram alterados. Discuta os efeitos biolgicos dessas mutaes se: a) o mutante tem
maior afinidade pelo sitio ativo; b) o mutante tem menor atividade pelo sitio ativo.
6) O nome da doena, diabetes mellitus, remete ao fato de que os pacientes produzem muita urina
(diabetes) adocicada (mellitus). Explique essas caractersticas.
7) Diabetes tipo I geralmente caracterizada por uma limitao significativa na liberao de insulina
pela clula pancretica. Por outro lado, pacientes sofrendo de diabetes tipo II normalmente
apresentam nveis normais, ou at ligeiramente elevados, de liberao de insulina. Como essas
diferenas refletem na elaborao de estratgias teraputicas?
8) Um dos genes epidemiologicamante associados com diabetes tipo II um fator de transcrio
chamado TCF7L2 (transcription factor 7like 2), que regula a transcrio do gene do proglucagon.
Discuta como um problema nesse gene pode estar associado ao desenvolvimento de diabetes tipo
II.
9) Explique a acidose sangunea de pacientes com diabetes no controlada.

99

METABOLISMO MUSCULAR

1. As principais fontes de energia para o msculo so a glicose proveniente da quebra do


glicognio, cidos graxos e corpos cetnicos. O msculo bem alimentado, no repouso,
sintetiza um estoque de glicognio que chega a representar de 1 a 2 % de sua massa. Tal
estoque serve como fonte de combustvel prontamente disponvel.
2. O msculo no disponibiliza glicose para outros tecidos, uma vez que ele carece de glicose6-fosfatase. Entretanto, ele serve como um reservatrio de energia porque, durante o jejum
prolongado, suas protenas so degradadas a aminocidos, muitos dos quais so
convertidos a piruvato que, por sua vez, transaminado a alanina. A alanina ento levada
ao fgado atravs da corrente sangunea, onde sofre transaminao para piruvato, um
precursor da glicose (ciclo da glicose-alanina).
3. Visto que o msculo no realiza gliconeognese, ele carece da maquinaria que controla esse
processo em rgos gliconeognicos como rins e fgado. No possui receptores para
glucagon. Contudo, possui receptores para epinefrina, que regula a quebra e sntese de
glicognio.
4. Os msculos cardaco e esqueltico possuem diferentes isozimas PFK-2/FBP-2. Enquanto a
isozima cardaca controlada por fosforilao (de modo oposto do fgado), a do msculo
esqueltico no o . Portanto, a concentrao de frutose-2,6-BP aumenta no msculo
cardaco, mas diminui no fgado em resposta a um aumento nos nveis de AMPc. Ademais, a
isoforma muscular da piruvato quinase no est sujeita fosforilao/desfosforilao como a
isoforma heptica. Logo, ao passo que um aumento de AMPc no fgado estimula a quebra de
glicognio e a gliconeognese, resultando em liberao de glicose, um aumento de AMPc no
miocrdio ativa a quebra de glicognio e a gliclise, resultando em consumo de glicose.
Conseqentemente, a epinefrina atua independentemente do glucagon que, atuando
reciprocamente com a insulina, regula os nveis sanguneos de glicose.
5. A contrao muscular depende da hidrlise do ATP e, conseqentemente, do processo de
respirao. O msculo esqueltico, no repouso, utiliza cerca de 30 % do oxignio consumido
pelo corpo humano. Durante uma carga de exerccio pesado, a hidrlise do ATP aumenta
muito mais do que a velocidade da respirao. O ATP inicialmente regenerado pela reao
do ADP com fosfocreatina, catalisada pela creatina quinase:
Fosfocreatina + ADP creatina + ATP
(a fosfocreatina ressintetizada no msculo, em repouso, pela reao inversa).
Sob condies de exerccio vigoroso, contudo, o suprimento de fosfocreatina insuficiente e
o msculo tem que contar com a gliclise da glicose-6P proveniente da quebra do glicognio
para a produo de ATP. A velocidade desse processo excede aquela do ciclo de Krebs e

100

fosforilao oxidativa. Assim, a maior parte da glicose degradada anaerobicamente a


lactato que, no ciclo de Cori, levado ao fgado pela corrente sangunea, onde
reconvertido glicose atravs da gliconeognese.
6. O corao um rgo predominantemente aerbico. rico em mitocndrias (elas chegam a
compreender mais de 40 % de seu espao citoplasmtico). Pode metabolizar cidos graxos,
corpos cetnicos, glicose, piruvato e lactato. Os cidos graxos representam o combustvel de
escolha para o msculo cardaco em repouso mas, sob a imposio de uma carga de
trabalho pesado, ele aumenta grandemente seu consumo de glicose, que proveniente em
maior parte de seu limitado estoque de glicognio.

Exerccios 27
1) Porque voc respira ofegante aps um perodo de exerccio moderado?
2) Os nveis plasmticos de lactato desidrogenase (LDH) podem servir como elemento
diagnstico e prognstico para avaliao de leso cardaca em situaes de hipxia.
Explique.
3) Faa um esquema dos ciclos de Cori e da glicose-alanina. Quais vantagens esses ciclos
representam para o msculo esqueltico?
4) O consumo de glicose pelo msculo esqueltico maior em condies aerbicas ou
anaerbicas? Justifique.
5) Enquanto o glicognio heptico utilizado para manter os nveis sanguneos de glicose
dentro de valores normais, o glicognio muscular utilizado como fonte de energia
prontamente disponvel. Em que fato se pauta essa diferena?
6) Um indivduo sedentrio, aps um perodo de exerccios fsicos intensos, sente cibras nas
pernas. Explique, resumidamente, os eventos bioqumicos que originaram as cibras.
7) Explique os papis do ATP e do on Ca2+ na contrao muscular.
8) Mutaes em um tipo de miosina expressa em clulas ciliadas do sistema auditivo so
associadas a patologias com perda de audio. Postule um possvel papel da miosina nesse
sistema que explique o efeito da mutao.

101

TRANSDUO DE SINAIS
1)

As clulas precisam responder ao seu ambiente, recebendo, processando e respondendo a

sinais externos.
2)

De modo geral, a transduo de sinais envolve um sinal, tambm denominado como

primeiro mensageiro, reconhecido por um receptor, convertido em outros sinais qumicos


intracelulares (transduo por mensageiros secundrios), que podem sofrer passos de
amplificao, e finalmente geram uma ou mais respostas. A maioria dos sinalizadores
extracelulares no capaz de atravessar a membrana plasmtica, e requer um receptor associado
membrana para transmitir o sinal para o ambiente intracelular.
3)

A associao receptor-ligante leva a alterao da estrutura do receptor, induzindo a atividade

de enzimas as quais catalisam a sntese de mensageiros secundrios como cAMP, cGMP,


inositol-3,4,5-trifosfato ou levam ao aumento das concentraes intracelulares de Ca2+. Os
mensageiros secundrios permitem tanto a difuso do sinal para vrios ambientes intracelulares
quanto a amplificao do sinal. Muitos mensageiros secundrios agem ativando protenas
quinases, que transferem grupos fosforila do ATP para resduos especficos de serina, treonina e
tirosina proticos, promovendo alterao de estrutura e funo de enzimas. Protenas fosfatases
fosfatases atuam em sentido contrrio.
4) Um exemplo de resposta que envolve segundo mensageiro e fosforilao protica promovida
por receptores de sete hlices transmembranares, responsveis pela transmisso de uma
variedade de sinais. So compostos por um nico polipeptdeo que atravessa a membrana sete
vezes. A ligao do receptor com o ligante induz mudana conformacional induzindo a transduo
de sinal. Como primeiro passo h ativao da protena G associada ao receptor. Na sua forma
inativa, a protena G um heterotrmero que liga GDP na sua subunidade , e a interao com o
receptor ativado induz a substituio de GDP por GTP e dissociao da subunidade das
subunidades e . Existem subtipos de subunidade G. A subunidade Gs, associado ao receptor
de adrenalina, ativa a adenilato ciclase interagindo com a superfcie que anteriormente se ligava
s outras subunidades da protena G. Esta interao ativa a adenilato ciclase, que passa a
sintetizar mais AMPc. A subunidade Gi possui atividade contrria, inibindo a adenilato ciclase e
produo de AMPc. A atividade da G continua at que a sua atividade intrnseca GTPsica
degrada o GTP e leva-a a se associar de novo com as outras subunidades da protena G. O AMPc
pode afetar mltiplos processos celulares atravs da ativao da protena quinase A, que catalisa
a fosforilao de diversas protenas. Como exemplos, promove a ativao da glicogenlise e
inibio da sntese de glicognio por fosfoforilao da glicognio fosforilase e sintase,
respectivamente.
5) Receptores acoplados a protena Gq podem agir tambm atravs da cascata do inositol trifosfato
(IP3). O IP3 formado, aem do diacilglicerol, a partir da clivagem do fosfatidilinositol bifosfato,

102

presente nas membranas celulares, pela fosfolipase C- ativada pelas subunidades da


protena Gq. O IP3 formado difunde pela clula e causa liberao de Ca2+ do retculo
endoplasmtico. A concentrao elevada de Ca2+ citoslica ento promove uma srie de
respostas como, por exemplo, contrao muscular, quebra de glicognio ou secreo. Alm de
gerar IP3, o diacilglicerol produzido pela atividade enzimtica da fosfolipase C- age como
segundo mensageiro. O diacilglicerol se liga e ativa protena quinase C, a qual fosforila serinas e
treoninas em protenas alvo, como, por exemplo, canais de Ca2+ na membrana citoplasmtica,
ativando-os.
6) O on Ca2+ um importante secundo messangeiro com atividade regulatria sobre muitas
protenas incluindo protena quinases e fosfatases, as quais, quando ativadas por ligao de Ca2+
fosforilam ou desfosforilam demais protenas. A concentrao mdia de Ca2+ citoslico livre [Ca2+]i
em torno de 10 - 100 nM, enquanto a concentrao extracelular na faixa de 2 milimolar.
Contudo, o clcio tambm acumulado em alguns compartimentos intracelulares como, por
exemplo, no reticulo endoplasmtico. Entrentanto, a ativao de receptores de hormnios ou de
canais inicos levam a alterao de [Ca2+]i no citossol. Tipicamente h aumento transiente de
[Ca2+]i como resultado do estimulao da clula pelo hormnio ou neurotransmissor. Entranto, o
influxo de clcio pelos canais localizados na membrana como tambm a mobilizao de estoques
de clcio intracelulares podem contribuir para alteraes de [Ca2+]i. Uma via para liberao de
Ca2+ a partir de estoques no retculo endoplasmtico (RE) fica ativada com a sntese de IP3, o
qual se liga nos seus receptores no RE, resultando na abertura de canais de Ca2+ localizados na
membrana do RE. H vrios mecanismos que mantm a concentrao citoslica de Ca2+ baixa e
contribuem para as caractersticas transientes de elevaes de [Ca2+]i, incluindo ATPases na
membrana plasmtica e retculo sarco(endo)plasmtico, o trocador Na+/Ca2+ da membrana
plasmtica e o uniporter de Ca2+ mitocondrial.
7. Ao contrrio de receptores acoplados a protena G, receptores de neurotransmissores, tambm
denominados canais inicos ativados na presena do ligante, diretamente permitem o fluxo de
ons pela membrana citoplasmtica. Quando ativados, os receptores sofrem uma mudana
estrutural, resultando na formao de um poro e permitindo o fluxo inico.
8. Receptores com atividade enzimtica intrnseca, como tirosina quinases e fosfatases,
representam uma nica ala transmembrnica e possuem dois domnios: o receptor, voltado para
o lado externo da membrana e a poro com atividade enzimtica, a enzima efetuadora, voltada
para o citoplasma. Os receptores, quando ativos por ligao dos seus ligantes, exercem as suas
atividades catalticas de fosforilao e desfosforilao e iniciam a transduo de sinal. Em alguns
casos, a ligao do ligante provoca a dimerizao do receptor e, em outros casos, como o
receptor de insulina, o receptor j est dimerizado na ausncia do ligante. Existe ainda um terceiro
caso em que a protena receptora e a enzima efetuadora so entidades fsicas separadas. A

103

ativao do receptor leva dimerizao das suas subunidades, e subsequentemente a interao


com uma tirosina quinase citoslica. Essa tirosina quinase ativada pela interao com o receptor
fosforila protenas alvo e d inicio a transduo de sinal.
9. Hormnios lipoflicos esterides e tireoidianos no possuem receptores na superfcie celular.
Estes hormnios esterides e tireoidianos penetram facilmente na clula atravs da membrana
celular e encontram seus receptores proticos no interior do citoplasma ou ncleo. Na forma de
complexo com protenas, eles adentram o ncleo para promover sntese especfica de mRNA de
enzimas regulatrias. Entretanto, a ao de hormnios esterides, como, por exemplo, a
cortisona, lenta, uma vez que regula a sntese de protenas Por outro lado, a transduo de sinal
induzida por receptores acoplados a protena G, tirosina quinases e fosfatases rpida, pois leva
a regulao imediata da atividade de protenas alvo por fosforilao e desfosforilao.
10. A transduo de sinal, coordenada por uma rede imensa de diferentes receptores e seus
ligantes, controla as atividade de protenas chave no metabolismo, e responda a estmulos do
ambiente onde o organismo se encontra. A transduo de sinal tambm direciona o
desenvolvimento do organismo e tambm envolvido em condies fisiopatolgicas, como
cncer.

Exerccios 28
1) Compare a ao hormonal do sistema glucagon/insulina com cortisona na regulao da
gliconeogense.
2) Identifique as diferenas na transduao de sinal por glucagon e insulina, levando a sntese e
degradao do glicognio.
3) A toxina do clera permanentemente ativa a subunidade Gs enquanto a toxina pertussis
leva a inibio da subunidade Gi. Quais so as conseqncias imediatas sobre a concentrao
de adenilato ciclase e o que voc espera referente a formao de AMPc na presena de um
inibidor de fosfoesterase?
4) Efeitos diferentes podem ser induzidos pelas vias idnticas de transduo de sinal, como, por
exemplo, a produo de inositol-trifosfato levando a mobilizao de clcio intracelular, levando a
vasodilatao quando clulas foram ativadas pelo peptdeo A e a vasoconstrio quando
estimuladas pelo peptdeo B. Explique como isso possvel?
5) Clcio um messageiro importante e a concentrao citoslica de clcio fica elevada durante
a sinalizao por hormnios e neurotransmissores. Explique como a clula consegue abaixar a
concentrao de clcio citoslico aps o estmulo.
6) Na juno neuro-muscular occore elevao da concentrao citosolica de clcio por ativao
com acetilcolina. Explique a via de sinalizao e o mecanismo da regulao de troponina e
tropomiosina, levando a contrao muscular.

104

Você também pode gostar